Site Loader

Содержание

Решение задач. Электрический ток | Частная школа. 8 класс

Конспект по физике для 8 класса «Решение задач по теме Электрический ток». Как решать задачи на нахождение силы тока в цепи. Как решать задачи на нахождение напряжения в цепи. Как решать задачи на закон Ома.

Конспекты по физике    Учебник физики    Тесты по физике


Решение задач по теме


Электрический ток
ЗАДАЧА 1.

Через нить накаливания лампочки от карманного фонарика за 2 мин проходит электрический заряд, равный 30 Кл. Определите силу тока в этой лампочке.

Запишем условие задачи и решим её.

Ответ: I = 250 мА.

 

ЗАДАЧА 2.

Электродвигатель включён в электрическую цепь с напряжением 24 В. Определите заряд, прошедший через электродвигатель, если при этом была совершена работа, равная 84 кДж.

Ответ: q = 3500 Кл.

 

ЗАДАЧА 3.

Определите силу тока в кипятильнике, включённом в сеть с напряжением 220 В, если сопротивление спирали составляет 55 Ом.

Ответ: I = 4 А.

 

ЗАДАЧА 4.

Какое напряжение нужно приложить к концам проводника сопротивлением 5 Ом, чтобы по проводнику пошёл ток с силой тока, равной 300 мА?

Ответ: U = 1,5 В.

 

ЗАДАЧА 5.

Определите сопротивление резистора, если за время 10 мин через него проходит заряд 200 Кл. Напряжение на концах резистора равно 6 В.

Ответ: R = 18 Ом.


 

ИТОГИ темы «Электрический ток»

  • Электрическим током называют упорядоченное движение заряженных частиц под действием электрического поля.
  • Сила тока — это физическая величина, которая показывает, какой электрический заряд проходит через поперечное сечение проводника за единицу времени.
  • Работу электрического поля, создающего электрический ток, называют работой тока.
  • Напряжение показывает, какую работу совершает электрическое поле при перемещении единичного электрического заряда из одной точки поля в другую.
  • Электрическое сопротивление характеризует свойство проводника препятствовать протеканию в нём электрического тока.
  • Закон Ома гласит: сила тока на участке цепи прямо пропорциональна напряжению на концах этого участка и обратно пропорциональна сопротивлению.

 


Вы смотрели Конспект по физике для 8 класса «Решение задач. Электрический ток»

.

Вернуться к Списку конспектов по физике (Оглавление).

Просмотров: 48 657

Заряд при постоянной силе тока

    В ходе кулонометрических измерений необходимо точно определить заряд, проходящий через электрод до появления сигнала конечной точки. Если опыт проводят в гальваностатических условиях (при постоянной силе тока), заряд определяют как произведение силы тока на точно измеренное время опыта. Если же ток во время опыта непрерывно изменяется, то удобнее использовать специальные кулонометры — счетчики, количества электричества. Электрохимические кулонометры основаны на законах Фарадея в них измеряют объем выделяющегося газа или ртути, который пропорционален количеству электричества. Существуют также электромеханические кулонометры. 
[c.387]

    Электрохимический способ. Во многих случаях богатую информацию дает электрохимическое исследование окисленной поверхности. Если металл, покрытый окисной пленкой, погрузить в рас-, твор, химически инертный относительно пленки, и включить катодную поляризацию при постоянной силе тока, то электрод зарядится до потенциала, при котором станет возможным восстановление окисла. Пока будет протекать восстановление потенциал должен сохранять постоянную или почти постоянную величину, так как электрические заряды расходуются на восстановление окисла. По окончании восстановления электрод снова зарядится и примет значение, при котором возможно восстановление водорода при данной 
[c.91]

    Емкостью при заряде Qз называют количество электричества, которое требуется израсходовать при заряде аккумулятора в данных условиях. Значение Qз при постоянной силе тока находят умножением зарядного тока на продолжительность заряда Т  [c.113]

    Благодаря процессам выделения кислорода и водорода, на кривой напряжения наблюдается некоторый подъем (площадка б). После отключения аккумулятора от зарядной сети напряжение падает вследствие исчезновения поляризации с прекращением протекания тока, а также вследствие выравнивания концентрации кислоты (отдых аккумулятора). Таким образом при заряде аккумуляторов постоянной силой тока необходимо время от времени повышать зарядное напряжение. Это достигается, например, путем уменьшения сопротивления реостата. 

[c.231]

    Заряд при постоянной силе тока [c.256]

    Заряд при постоянной силе тока (рис. 7.2, а). При этом способе значение зарядного тока в течение всего времени заряда остается постоянным. Этот способ является основным и наиболее универсальным. Заряжаемые батареи соединяют последовательно между собой. Последовательно с ними включают и реостат, с помощью которого регулируют силу зарядного тока для этой цели применяются и другие регуляторы, например тиристорные, которые, периодически включая и выключая сопротивление в цепи, меняют значение тока так, что среднее его значение оставалось постоянным во времени. Число одновременно включенных на заряд батарей зависит от напряжения сети Ус, к которой подключается группа батарей. Для полного заряда свинцово-кислот-ной аккумуляторной батареи требуется напряжение 

[c.96]


    Никель-кадмиевые аккумуляторы могут заряжаться при постоянной силе тока, при постоянном потенциале и при модифицированном постоянном потенциале. Выделение газа начинается примерно при 1,47 в, обычно в конце 4,5-часового периода при режиме заряда, рассчитанном на его завершение в течение 7 ч. Конечное напряжение 1,75 в на элемент, требуемое напряжение источника тока 1,85 в на элемент. 
[c.298]

    Как правило, применяется заряд при постоянной силе тока реже при постоянном напряжении и постепенно уменьшающейся силе тока. [c.749]

    Если минерал, покрытый оксидной пленкой, погрузить в раствор, химически инертный относительно пленки, и включить катодную поляризацию при постоянной силе тока, то электрод приобретет значение потенциала, при котором становится возможным восстановление окисленного соединения. Пока будет протекать восстановление, потенциал электрода будет практически неизменным, так как электрические заряды расходуются на восстановление окисленного соединения. Проведя подобные исследо- 

[c.123]

    Для определения плотности зарядов статического электричества используют гальванометры постоянного тока (микроамперметры, гальванометрические и электрометрические усилители). Шкалы этих приборов рассчитаны на измерение тока силой 20—30 мА и ниже. Высокочувствительные гальванометры позволяют измерить силу тока 10 —10 А. [c.176]

    Аналитическим стандартом в кулонометрии является постоянная Фарадея. Она представляет собой произведение заряда электрона (1,602-10″ Кл) на число Авогадро (6,022-10 моль ) и равна 96487 Кл/моль. Измеряя силу тока или количество электричества, можно установить, какое количество вещества вступило в реакцию на электроде, если, конечно, эта реакция является стехиометрической. 

[c.516]

    Отрицательные пластины получаются с более развернутой поверхностью и, следовательно, с большей емкостью в случае формирования при более низких температурах, а положительные пластины становятся более прочными при формировании в электролите с более высокой температурой. Однако колебания температуры допустимы только в определенных пределах ниже 10 и выше 60 С получается много брака. В летнее время к концу формирования электролит иногда требуется охладить. Начинают формирование при плотности тока 87—100 А/м , через 10 ч плотность тока снижают до 60—70 А/м и продолжают формирование еще 5—10 ч. Иногда после первой ступени формирования делают паузу в 1 ч. На некоторых предприятиях формирование производят при постоянном напряжении на баке. В этом случае в начале формирования плотность тока достигает 250—280 А/м и падает по мере заряда пластин до 60—70 А/м . Конец формирования определяют по появлению обильного газовыделения, постоянству напряжения на баках в пределах 2,6—2,75 В и по отсутствию на пластинах белых несмываемых пятен сульфата свинца. В процессе формирования следует контролировать силу тока, температуру и концентрацию 

[c.377]

    В кулонометрическом титровании бром генерируется п зНи электролитически в процессе титрования пропускаемым через раствор постоянным электрическим током. При этом измеряется время, за которое будет достигнута конечная точка титрования. По известным силе тока I и времени титрования / можно вычислить заряд Q в кулонах, прошедший через раствор за время титрования  

[c.61]

    При приложении к электродам постоянного напряжения V на границе металл-электролит образуется двойной электрический слой, в пределах которого протекают основные электрохимические процессы. Данный слой рассматривают как плоский конденсатор, обкладками которого являются поверхность электрода и слой ионов, расположенных вблизи поверхности электрода и имеющих противоположный знак заряда. По мере прохождения тока одного направления ионы, соприкасаясь с электродами, разряжаются и выделяются на них в виде атомов. Это приводит к постоянному уменьшению силы тока через раствор, что рассматривается как заряд конденсатора, образованного двойными электрическими слоями. Описанное негативное явление называют поляризацией электродов. Оно приводит к нелинейности вольт-амперной характеристики ячейки (рис. 6.6, б). 

[c.514]

    Электронно-дырочные инжекционные токи в полимерных диэлектриках исследованы значительно менее подробно по сравнению с ионными. Тем не менее, данные по таким токам для ряда полимеров [59, 60] позволяют сделать ряд важных заключений. Получить экспериментальную кривую / от т в режиме непрерывной инжекции с заметно выраженным максимумом тока представляется весьма затруднительным из-за плавного спада тока со временем даже при т Тп, что, возможно, вызвано наличием в полимерных диэлектриках глубоких ловушек для электронов, а это, в свою очередь, может быть связано [22, с. 103] со структурной неупорядоченностью (дефектами) полимеров. Поэтому для определения Тп и расчета у. по формуле (101) часто используют так называемый импульсный метод. Ионизирующим излучением в тонком приэлектродном слое б носители заряда в течение т Тп- Под действием приложенного напряжения тонкий слой заряда движется по нанравлению к коллекторному электроду, что вызывает появление имиульса тока, сила которого резко уменьшается до уровня темпового тока, когда движущийся заряженный слой достигает противоположного электрода, т. е. в момент времени т = Тп- Если сила тока при О [c.78]


    Вернемся к историческому опыту Зеебека и рассмотрим его с позиций сегодняшнего дня. Так же, как и в случае электрической цепи Пельтье, развернем спаи цепи в рабочие поверхности термоэлемента Зеебека. Порции тепловой энергии от источника теплоты поступают на горячий спай термоэлемента (см. рис. 10). Дрейфовая подвижность свободных электронов в области горячего спая увеличивается из-за их избыточной кинетической энергии (и, соответственно, скорости). При этом на фоне броуновского движения свободных электронов во всем объеме ветвей термоэлемента возникает преимущественный их дрейф в область холодного спая. Избыток отрицательного заряда в области холодного спая и положительного (ионы решетки) в области горячего спая вызывает появление внутреннего тормозящего электрического поля, препятствующего дальнейшему движению свободных носителей на холодный спай. Таким образом, термоэлемент. Зеебека с разомкнутой электрической цепью превращается (при наличии источника теплоты) в заряженный твердотельный аккумулятор , между полюсами которого существуют электрическое поле и свободные заряды с разным электрическим потенциалом. При замыкании электрической цепи с термоэлементом Зеебека в ней возникает постоянный электрический ток. Источником тока, совершающим работу внешних сил по разделению электрических зарядов, является тепловая энергия источника тепла. [c.32]

    Катионы и анионы. Если через раствор (или расплав), содержащий достаточно подвижные ионы, протекает постоянный электрический ток (при погружении в раствор двух электродов), то ионы приобретают определенным образом направленное движение в отсутствие тока ионы перемещаются хаотично (рис. 59). Вследствие электростатических сил притяжения ионы перемещаются к тому электроду, который обладает противоположным им по знаку зарядом. [c.167]

    Полное окончание заряда обычно можно установить, продолжая заряд постоянной силой тока до тех пор, пока напряжение на зажимах батареи не достигнет максимума. Так как этот момент 1Может быть определен лишь в том случае, если заряд продолжается и после того, как этот максимум достигнут, то, относя коэффициент отдачи к полной величине заряда, мы получаем значение для него, меньшее действительного. Числом ампер-часов, полученных батареей после достижения максимума, можно пренебречь, но в некоторых случаях погрешность в определении отдачи, если она производится по данным одного цикла заряд—разряд, может оказаться очень большой. Поэтому на практике определение коэффициента отдачи обычно основывается на ряде последовательных циклов, [c.363]

    Оба варианта детектора ( классический ДЭЗ и ДПР) в конечном счете имеют общий механизм образования сигнала, сводящийся к уменьшению электрической проводимости (увеличению сопротивления) газового промежутка между электродами детектора за счет связывания свободных электронов молекулами электроноакцепторных веществ. При этом в ДЭЗ фиксируется уменьшение силы тока при постоянном напряжении, а в ДПР — увеличение разности потенциалов на электродах при постоянной силе тока детектора. Вместе с тем детектор постоянной скорости рекомбинации обладает рядом существенных преимуществ перед ДЭЗ, среди которых следует назвать в первую очередь значительное расширение линейного динамического диапазона по сравнению с той же конструкцией в режиме измерения силы тока. Это достигается как за счет увеличения верхнего предела концентраций, так и за счет снижения предела детектирования, который для ДПР доведен до значения, не превышающего 10 мг/см по 7-гексахлорцнклогексану. Весьма важно также, что повышение напряженности поля при введении анализируемого вещества в ДПР препятствует образованию объемного заряда и устраняет влияние контактной разности потенциалов на процессы сбора заряженных частиц, те.м самым обеспечивая большую устойчивость работы детектора и отсутствие искажений сигнала. [c.127]

    При нормальной эксплуатации основными операциями по уходу за аккумуляторами являются их регулярный подзаряд, доливка дистиллированной воды, а также периодическая замена электролита. Заряд свинцовых аккумуляторов можно осуществлять при постоянной силе тока, численно равной примерно 10% номинальной емкости батареи, при этом выгоднее заряжать аккумулятор при силе тока вдвое меньшей после достижения напряжения 2,3—2,4 В. Щелочные аккумуляторы заряжают один раз в месяц нормальным зарядным током в течение 12 ч и дополнительно в течение 6 ч током, уменьшенным в два раза. СЦ аккумуляторы заряжают током, численно равным 10—177о номинальной емкости, не допуская увеличения напряжения свыше 2,1 В. [c.284]

    Заряд аккумуляторов можно проводить при постоянной силе тока, численно равной 0,1 номинальной емкости батареи. Выгодней заряд проводить, снижая силу тока наполовину после достижения напряжения 2,3—2,4 В. На автомобилях заряд проводится прн постоянном напряжении, поддерживаемом реле-регулятором. Правильная регулировка реле крайне важна, так как при завышенном напряжении приходится постоянно доливать воду в аккумуляторы, и их срок службы резко снижается. Слишком низкое напряжение вызовет недозаряд аккумуляторов. При длительном хранении аккумуляторов с электролитом без употребления 1 раз в месяц им след>ь-ет давать заряд до обильного газовыделения и 1 раз в 3 месяца глубокий разряд и полный заряд. Хранить аккумуляторы с электролитом разряженными нельзя. Не рекомендуется также хранить аккумуляторы, бывшие в употреблении, с вылитым электролитом. [c.380]

    Основным показателе.м ХИТ является разрядная кривая — зависи,мость напряжения от количества пропущенного электричества Q или, при разряде постояннее силой тока, от времени. Для акку. улятора характеристикой является и аналогичная зарядная кривая. Типичные зарядные и разрядные кривые для свинцового акку.мулятора представлены на рнс. 16.1. По мере разряда напряжение падает (общее перенапряжение элемента растет). Разряд проводят,до определенного конечного напряжения екон-Общее количество электричества, которое можно получить до достижения этого напряжения, называют разрядной е.мкостью данного ХИТ. Произведение емкости на среднее разрядное напряжение—энергозапас данного ХИТ. Основными эксплуатационными показателями ХИТ являются удельная энергия на единицу массы или объема, максимальная удельная. мощность, сохраняемость (для первичных элементов), ресурс— допустимое число зарядно-разрядных циклов, а также коэффициент полезного действия по энергии — отношение энергии, полученной прн разряде и затраченной при заряде (для аккумуляторов), срок службы, температурный интервал работоспособности, механическая прочность, невыливаемость электролита и г. д. [c.308]

    Из кривых заряда и разряда аккумулятора можно вычислить к. п. д. аккумулятора по энергии и по току. Действительно (рис. 6), при постоянной силе тока площадь ММСВ соответствует количеству электрической энергии, полученной от аккумулятора, а площадь MNDA — энергии, поглощенной аккумулятором. [c.40]

    Заряд Ц-С аккумуляторов производится при постоянной силе тока и прекращается по достижении напряжения 2,05—2,10 в. Если при этом заряд не прекратить, то начинается выделение кислорода, разрушающего сепарацию. При тюрмальных заряде и разряде (при нагреве электролита не выще 80°С) отдача по току приближается к 100%, а по энергии — к 85%. Саморазряд Ц-С аккумуляторов незначителен и достигает за 6 месяцев хранения не более 30% емкости. Срок службы, по данным фирмы Уеппег , исчисляется несколькими сотнями циклов. Срок службы стартерных типов исчисляется несколькими десятками циклов. Измене 1ие температуры заметно сказывае тся на емкости Ц-С аккумуляторов. Понижение температуры приводит к заметному снижению емкости, поэтому эксплуа- [c.528]

    Типичная кривая заряжения, полученная быстрым гальваноста-тическим методом, представлена на рис. 1. На кривой имеются две задержки, т. е. области с высоким значением псевдоемкости, в которых скорость изменения потенциала во времени при постоянной силе тока относительно мала. Первая задержка соответствует ионизации поверхностных атомов водорода, а вторая — образованию поверхностного окисла или адсорбционного слоя кислорода. Между двумя задержками имеется область с низким значением емкости (в которой потенциал быстро изменяется во времени), которая отвечает изменению заряда и строения ионного двойного слоя. [c.397]

    Заряд аккумуляторов можно вести четырьмя способами 1) при постоянном напряжении, 2) цри постоянной силе тока, 3) ступенчатым зарядом и 4) модифицированным споообом. [c.255]

    Гкден Ие зарядной силы тока, как это следует из уравнений, вызывается тем, что э. д. с. аккумулятора и его внутреннее сопротивление с течением времени заряда постепенно увеличиваются. Поэтому заряд аккумуляторов на полную емкость при таком способе длится очень долгое время. Положительной стороной способа шляетс то, что В нем отсутствует потеря энергии в ре01стате, которая имеет место во врем заряда при постоянной силе тока. а На рис. 52 графиче- [c.256]

    Порядок проведения КТЦ заряжают аккумуляторные батареи при постоянной силе тока /з = 0,1 С20 А до напряжения не менее 2,4 В на каждом аккумуляторе батареи, а затем снижают ток до значения /з=0,05СгоА и ведут процесс до полного заряда батареи. Контрольный разряд проводится постоянным током /р=0,05 С20 А до конечного напряжения на аккумуляторе, равном [c.103]

    Для заряда можно пользоваться только постоянным током. Если в распоряжении имеется лишь переменный ток, то он должен быть преобразован в постоянный. Это может быть осушествлено при помоши синхронного лреобразовате-ля, мотор-генератора или выпрямителя. Общеприняты две системы заряда а) при постоянной силе тока б) при постоянном потенциале или постоянном апряжении. Этот [c.265]

    Для исправления сульфатированвьих пластин предлагались разли чные средства. Однако самым простым и действенным средством в таком случае будет следующая обработка выливают электролит из элемента и наполняют его водой после того как батарея простоит так около часа, ее можно начать заряжать слабым током при з словии, что напряжение на зажимах элементов будет меньше 2,3 в на элемент. Сопротивление батареи вначале будет высоким и, следовательно, ток очень слабым, но если напряжение на зажимах будет поддерживаться постоянным, то сила тока будет возрастать с одновременным уменьшением количества сульфата. В этих условиях ток будет использоваться в той мере, в какой элементы способны подвергаться заряду процесс будет становиться более или менее автоматическим, нужно только следить за температурой и выключать батарею или ослаблять ток, как только температура достигает 43° С. Заряд можно производить также. методом постоянной силы тока, но при условии малой его величины. По мере заряда элементов налитая в них вода превращается в раствор серной кислоты и становится возможным вести наблюдения за возрастанием удельного веса. Если конечный удельный вес после продолжительного заряда остается неизменным, не достигая, однако, требуемой величины , то следует добавить электролит. Нередко случается, что удельный вес электролита после заряда в чистой воде поднимается выше нормального, например до 1,280. Это служит явным доказательством того, что в свое время в элементы добавлялась кислота тогда, как этого не следовало делать и когда они нуждались только в воде. [c.339]

    Когда замыкающие глицинатные ионы доходят до рабочего геля, их число и подвижность увеличиваются, поскольку значение pH 8,9 приближается к рКа. глицина. Теперь подвижность глицинатных ионов становится выше, чем у белковых анионов. Глицинатные ионы подтягиваются , при этом образуется более резкая граница между ними и ионами хлора. С белковыми анионами такого не происходит, поскольку молекулярное сито при переходе от верхнего геля к рабочему становится более мелким. Как только последний ион хлора уходит из рабочего геля, pH повышается, потому что ионы хлора заменяются более основными глицинатными, и вместо исходной системы трис-НС1 образуется трис-глициновый буфер. Вследствие этого возрастает отрицательный заряд белковых анионов, и их движение в однородном электрическом поле происходит в соответствии с величинами отношения заряд/масса Электрофорез обычно завершается в пределах 1 ч при постоянной силе тока 10 мА на одну рабочую трубочку, [c.133]

    Заряды статического электричества могут накапливаться на людях. Это наблюдается в тех случаях, когда пользуются обувью с подошвами, не проводящими электричество, носят одежду и белье из шерсти или искусственных волокон, находятся на полу, не проводящем электричество, и постоянно выпшгняют операции е диэлектриками. Действие статического электричества на человека проявляется своеобразными уколами и толчками, иногда сильными, однако они непосредственно не опасны, так как сила тока зарядов здесь очень мала (выражается в миллиамперах). Но при внезапном уколе человек может рефлекторно сделать непроизвольное движение и попасть в неогражденные движущиеся части машины или упасть с высоты. Такие случаи бывали. Есть данные о том, что длительное воздействие статического электричества неблагоприятно отражается на здоровье работающего и на его настроении, [c.46]

    Далее устанавливают градуированные капилляры и проверяют герметичность ячейки. Если положение менисков жидкости в капиллярах не изменяется в течение 3—5 мин, это показывает, что прибор герметичен. Прибор подключают к источнику постоянного тока, включают тумблер сеть и по секундомеру измеряют время прохождени5[ мениска жидкости между делениями капилляра. По направлению д[и жe-ния жидкости через мембрану к тому или иному электроду определяют знак заряда частиц. Кроме того, по миллиамперметру фиксируют значение силы тока. Затем тумблер сеть выключают, изменяют полярность электродов переключателем полярности и снова проводят измерение. [c.98]

    Здесь Т1 — квантовый выход фотокатода (1 — — Я) — постоянная, определяемая конструктивными особенностями ФЭУ (обычно 1,5ч-3,0) /т —сила тока термоэмиссии Д/ — полоса пропускапия усилителя е — заряд электрона. [c.80]

    Ионы, существующие в растворе электролита, испытывают различные воздействия со стороны окружающих частиц и соверщают постоянные перемещения, которые в отсутствие внешнего электрического поля имеют хаотичный характер. Наложение электрического поля приводит к появлению действующих на ионы электрических сил, которые имеют определенное направление. В результате возникает преимущественное перемещение (миграция) положительных ионов к отрицательному электроду, а отрицательных ионов — к положительному. Это обеспечивает перенос электрических зарядов. Возникает электрический ток, величина которого зависит от заряда ионов, их размера, характера сольватации и других взаимодействий с окружающими частицами, что, очевидно, связано с природой электролита и растворителя, а также с концентрацией раствора. Кроме того, величина электрического тока зависит от приложенного напряжения, геометрического расположения и размеров электродов, которые непосредственно влияют на напряженность возникающего электрического поля, а следовательно, и на скорость направленного движения ионов. Средняя скорость упорядоченного движения и данного типа ионов, отнесенная к напряженности действующего электрического поля Е, называется подвижностью (иногда абсолютной скоростью) иона и = ь/Е и определяется лишь природой и концентрацией раствора, а от величины электрического поля не зависит. В поле с напряженностью = 1 В-см числовые значения и к V совпадают. [c.216]

    Советским электрохимикам удалось создать тонкую экспериментальную методику исследования электродных процессов оо-строение поляризационных кривых в стационарных и нестационарных условиях, метод с использованием переменных токов, ос-циллографический метод, позволяющий установить временную зависимость потенциала электрода при пропускании тока постоянной силы, метод меченых атомов и др. Новые инструментальные методы раскрыли перед исследавателями более широкие горизонты. Так, было показано, что основным фактором, определяющим возникновение скачка потенциала на границе между металлом и раствором, является двойной электрический слой из зарядов металла и ионов раствора. Было найдено, что на условия появления и величину скачка потенциала между металлом и раствором большое влияние оказывает адсорбция и ориентация дипольных молекул. Сопоставление данных, полученных при изучении электрокапиллярных я влений, пролило яркий свет на роль поверхностно активных и коллоидных веществ, адсорбирующихся на поверхности электродов. [c.3]

    Электронный стрелочный компенсатор ЭСК-1 применяется для измерений напряжения и тока при электроразведочных работах методами постоянного тока (ВЭЗ, электропрофилирование, метод заряда, метод естественного ноля и т. д.). Прибор позволяет компенсировать э. д. с. поляри.зации, измерять разность потенциалов на электродах MN при отсутствии и наличии тока в линии АВ, измерять силу тока в цепи АВ. [c.112]

    Действие электрофильтра основано на ионизации молекул газового потока, проходящего между двумя электродами, к которым подведен постоянный электрический ток. Основные элементы электрофильтра -коронируюище и осадительные электроды. Отрицательное напряжение обычно подводят к коронируюшему электроду, а положительное — к осадительному. Поэтому к осадительным электродам под действием разности потенциатов движутся только отрицательные ионы и свободные электроны. Последние на своем пути сталкиваются со взвешенными в газовом потоке мелкими твердыми или 5КИДКИМИ частицами, передают им отрицательные заряды и увлекают к осадительным электродам. Подойдя к осадительному электроду, частицы пыли или тумана оседают на нем, разряжаются и при встряхивании отрываются от электрода под действием собственной силы тяжести. [c.12]


Самостоятельная работа «Сила тока. Напряжение» 8 класс

Самостоятельная работа «Сила тока. Напряжение»

Вариант №1

1.      Через проводник за 20 минут протекает заряд 2000 Кл электричества. Определите силу тока в проводнике.

2.      Сила тока в утюге 0.2 А. Какой электрический заряд пройдет через спираль за 5 минут?

3.      При электросварке сила тока достигает 200 А. За какое время через поперечное сечение электрода проходит заряд 60000 Кл?

4.      Чему равно напряжение на участке цепи, на котором электрическое поле совершило работу 0.5кДж при прохождении заряда 25Кл?

5.      Напряжение на лампочке 220 В. Какую работу совершает электрическое поле при прохождении через нить накала лампочки заряда 7 Кл?

6.      Напряжение на автомобильной лампочке 12 В. Какой заряд прошел через нить накала лампочки, если при этом была совершена работа 1,2 кДж?

7.      Амперметр показывает значение силы тока в проводнике 6 А в течение 1.5 минуты. Какая работа совершена в этом проводнике электрическим током, если напряжение соответствует 14 В?

8.      В течение 5 минут по цепи протекал ток в 5 А. Под каким напряжением находилась цепь, если в ней совершена работа 20,8 кДж?

 

________________________________________________________________________________

           

Самостоятельная работа «Сила тока. Напряжение»

Вариант №2

1.      Через спираль  электроплитки за 2 минуты прошел заряд в 600 Кл. Определите силу тока в спирали.

2.      Какой электрический заряд пройдет за 3 минуты через амперметр при силе тока в цепи  0.2А?

3.      За какое время через поперечное сечение проводника пройдет заряд, равный 30 Кл, при силе тока 200 мА?

4.      При прохождении по проводнику электрического заряда12 Кл совершается работа 0.6 кДж. Чему равно напряжение на концах этого проводника?

5.      Вычислите работу, которая совершается при прохождении через спираль электроплитки заряда 15 Кл, если она включена в сеть с напряжением 220 В?

6.      Напряжение на лампе накаливания 220 В. Какой заряд прошел через нить накала лампы, если при этом была совершена работа 4.4 кДж?

7.      Вольтметр, присоединенный к концам проводника, показывает 15 В. Работа, совершенная электрическим током в этом проводнике, составляет 240 Дж. Сколько времени по цепи протекал электрический ток, если амперметр, включенный в эту цепь, показал силу тока 1.5 А?

8.      Работа электрического тока в цепи равна 2.4 кДж. Напряжение на концах цепи составляет 8 В. Чему равна сила тока в цепи, если время его протекания 6 минут?

Самостоятельная работа «Сила тока. Напряжение»

Вариант №3.

1.      Определите силу тока в электрической лампочке, если через ее нить накала за 10 минут проходит электрический заряд 300 Кл.

2.      Сила тока, идущего по проводнику, равна 2 А. Какой заряд проходит по проводнику за 10 минут?

3.      Сколько времени длится молния, если через поперечное сечение ее канала протекает заряд 30 Кл, а ток равен 25000 А?

4.      На участке цепи совершена работа 3 Дж при прохождении по нему заряда, равного 0.2 Кл. Каково напряжение на этом участке цепи?

5.      Напряжение на электроприборе 100В. Какая совершена в нем работа, если прошел заряд, равный 0.5 Кл?

6.      Сила тока в цепи 2.5 А в течение 6 минут. Чему равно напряжение на концах цепи, если работа, совершенная электрическим током за это время, равна 14.4 кДж?

7.      Напряжение на концах электрической цепи равно 10 В, а сила тока 4 А. Какое время протекал ток по цепи, если работа, совершенная им, равна 28.8 кДж? Ответ выразите в минутах.

8.      В течение 11 минут через лампочку протекает электрический ток, значение которого — 110 мА. Электрическое напряжение на этом участке — 3,76 В. Найди работу электрического поля по перемещению заряда, произведённую за это время.

 

_________________________________________________________________________

Самостоятельная работа «Сила тока. Напряжение»

Вариант №4.

1.      Какова сила тока в цепи, если в течение 4 минут сквозь ее поперечное сечение прошел заряд 120 Кл?

2.      Время разряда молнии равно 3 мс. Сила тока в канале молнии около 30 кА. Какой заряд проходит по каналу молнии?

3.      При электросварке сила тока достигает 210А. За какое время через поперечное сечение электрода проходит заряд 64000Кл?  

4.      Определите напряжение на концах участка цепи, если по нему прошло 30 Кл электричества и при этом совершена работа 4.2 кДж.

5.      В собранной электрической цепи напряжение на одном из участков равно 12 В. Какая работа совершается электрическим током, если по цепи протекает 28 Кл электричества?

6.      Сколько кулонов электричества прошло по спирали электрической плитки, включенной в сеть с напряжением 220 В, если ток совершил работу 3.3 кДж?

7.      В течение 9 минут через лампочку протекает электрический ток, значение которого — 243 мА. Электрическое напряжение на этом участке — 7,7 В.  Найди работу электрического поля по перемещению заряда, произведённую за это время.

8.      Работа электрического тока в цепи равна 2.8 кДж. Напряжение на концах цепи составляет 9 В. Чему равна сила тока в цепи, если время его протекания 12 минут?

Самостоятельная работа «Сила тока. Напряжение»

Вариант №5.

1.      Через электрическую плитку за 10 минут протекает 3000 Кл электричества. Определите силу тока в  плитке.

2.      Ток в электронагревательном приборе 5 А. Чему равен заряд, который пройдет через нагреватель за 3 минуты?

3.      За какое время через поперечное сечение проводника пройдет заряд, равный 60 Кл, при силе тока 180 мА?

4.      Под каким напряжением находится лампочка, если через ее нить прошло 300 Кл и выделилось 1.5 кДж теплоты?

5.      Напряжение на электрической лампе 6 В. Какая работа совершается при прохождении через поперечное сечение нити накаливания этой лампы 12 Кл электричества?

6.      Какой заряд перемещается на участке цепи, если при этом совершается работа 108,88 Дж? Электрическое напряжение на участке цепи — 19 В.

7.      Вольтметр, присоединенный к концам проводника, показывает 14 В. Работа, совершенная электрическим током в этом проводнике, составляет 210 Дж. Сколько времени по цепи протекал электрический ток, если амперметр, включенный в эту цепь, показал силу тока 3 А?

8.      Работа электрического тока в цепи равна 3.2 кДж. Напряжение на концах цепи составляет 10 В. Чему равна сила тока в цепи, если время его протекания 9 минут?

_____________________________________________________________________________

            Самостоятельная работа «Сила тока. Напряжение»

Вариант №6.

1.      Через радиоприемник прошло 900 Кл электричества. Как велик был ток, если передача длилась 20 минут?

2.      В проводнике, включенном в цепь на 2 минуты, сила тока была равна 700мА. Какое количество электричества прошло через его сечение за это время?

3.      При электросварке сила тока достигает 260 А. За какое время через поперечное сечение электрода проходит заряд 65800 Кл?

4.      Чему равно напряжение на участке цепи, на котором совершен работа 1,5кДж, при прохождении заряда 30Кл?

5.      Напряжение на лампочке 120 В. Какая работа совершается электрическим током при прохождении по цепи 500 Кл электричества?

6.      Какой заряд проходит по участку цепи, если при напряжении на концах участка 24В работа тока в нем равна 96 Дж?

7.      Напряжение на концах электрической цепи равно 12 В, а сила тока 5 А. Какое время протекал ток по цепи, если работа, совершенная им, равна 24.6 кДж? Ответ выразите в минутах.

8.      В течение 13 минут через лампочку протекает электрический ток, значение которого — 160 мА. Электрическое напряжение на этом участке — 5,67 В. Найди работу электрического поля по перемещению заряда, произведённую за это время.


 

9.      Скачано с www.znanio.ru

Электрический ток и условия его существования

Нам известно, что телу можно сообщить заряд. Если не прикасаться после этого к телу, то полученный заряд будет оставаться на этом теле, то есть, перемещаться не будет.

Но если заставить заряд двигаться, можно наблюдать интересные явления. Потому, что именно движущиеся заряды создают:

  1. выполнение работы электрическими двигателями;
  2. выделение тепла в электрических нагревательных приборах;
  3. свечение ламп накаливания.

Рис. 1. Что происходит благодаря движению зарядов

Скорость теплового движения свободных электронов

Нам известно, что общий заряд тела состоит из большого количества элементарных зарядов.

К примеру, в твердых телах положительные заряды – это ядра атомов, или ионы. А отрицательные – это электроны.

А в жидкостях или газах – положительные и отрицательные заряды – это ионы.

Примечание: Ион – атом, у которого присутствует избыток электронов, либо наоборот, электронов меньше, чем в нейтральном атоме.

Рассмотрим твердый проводник, в нем присутствуют свободные заряды. Это такие электроны, которые оторвались от своего атома и свободно путешествуют по всему объему проводника.

Рис. 2. Отличия свободных и связанных электронов в проводнике

Примечание: Проводник – это тело, в котором много свободных электронов.

Как известно из молекулярно-кинетической теории (МКТ), мельчайшие частицы вещества находятся в непрерывном движении. Это движение возникает под действием температуры, поэтому, его часто называют тепловым. Такое движение беспорядочное, то есть — хаотическое.

Рис. 3. Под действием температуры свободные заряды беспорядочно движутся

Рассчитаем, с какой скоростью электроны в проводнике беспорядочно перемещаются под действием температуры.

Для этого воспользуемся формулой среднеквадратичной скорости частиц из молекулярной физики:

\[\large v = \sqrt{\frac{3kT}{m}}\]

Подставим в формулу такие числовые значения:

\(\large T = 300 \left( K\right)\) – комнатная температура +27 градусов Цельсия;

\(\large k = 1,38 \cdot 10^{-23} \left( \frac{\text{Дж}}{K}\right) \) – постоянная Больцмана;

\(\large m = 9,1 \cdot 10^{-31} \left(\text{кг}\right) \) – масса электрона;

После расчетов получим скорость, примерно равную

\[\large v \approx 117 \left(\frac{\text{км}}{\text{сек}}\right) \]

Как видите, это очень большая скорость, более 100 километров в секунду.

Рис. 4. Скорость свободных электронов в меди

Примечание: Физики свободные электроны в проводнике рассматривают, как частицы идеального газа. Его так и называют – электронный газ.

Однако, еще раз подчеркну, что тепловое движение – хаотическое. С помощью такого движения электрический ток не создать. Потому, что ток – это направленное движение зарядов.

Что такое электрический ток

Электрический ток – это направленное движение электрических зарядов.

В металлических проводниках движутся отрицательные заряды — электроны.

А в других проводниках, например, в жидких электролитах, направленно могут двигаться положительные и отрицательные ионы.

Внутри полупроводников заряд переносят электроны и дырки.

Примечание: Дырка – это псевдочастица, вакантное место для электрона. Она имеет положительный заряд, ее можно рассматривать, как пузырек, находящийся в электронном газе.

Рис. 5. В различных средах электрический ток создают такие заряды

Мы видим, что электрический ток может создаваться движением, как положительных частиц, так и отрицательных.

При этом, положительные частицы будут притягиваться к отрицательному полюсу источника тока и двигаться по цепи к нему.

А отрицательные частицы будут притягиваться и двигаться к положительному полюсу источника тока.

Примечание: Чтобы определить направление движения заряженных частиц, можно воспользоваться аналогией с течением воды: Заряды, как вода, движутся оттуда, где их много, туда, где их мало. На заре изучения электричества считали, что во время протекания тока в телах протекает некая электрическая жидкость. Поэтому для электрического тока применяется аналогия с течением воды. Позже выяснилось, что никакой электрической жидкости в телах нет.

Если заряды движутся направленно, значит, и ток будет иметь направление.

Куда направлен ток

Как выбрать направление электрического тока? На движение каких частиц – положительных, или отрицательных, ориентироваться? Оказывается, направление тока — это условный выбор.

Физики договорились, что направление электрического тока совпадает с направлением движения положительных зарядов. Значит, ток направлен от «+» к «-» выводу источника тока.

Пусть, известно направление вектора напряженности \(\large \vec{E} \) электрического поля. Чтобы определить направление тока, нужно считать, что в этом поле движутся положительно заряженные частицы.

Положительные заряды будут двигаться по направлению вектора \(\large \vec{E} \), а отрицательные – навстречу вектору.

Рис. 6. Куда направлен ток

Примечание: В металлах электроны движутся от минуса к плюсу, а ток направлен от плюса к минусу

Рис. 7. Направление движения зарядов и вектора напряженности электрического поля

Примечание: Наличие направленного движения зарядов можно определить косвенно. Протекая по проводнику, ток воздействует на этот проводник. Известны тепловое, химическое, или магнитное действие тока.

Чем больше ток, то есть, чем он сильнее, тем более заметно его действие.

Что такое поперечное сечение проводника

Электрический ток – это направленно движущиеся по проводнику свободные заряды. Его можно определить, когда известно количество заряженных частиц, прошедших через проводник.

Проводник может быть достаточно длинным. Поэтому неудобно учитывать заряды, находящиеся во всей длине проводника.

Чтобы было проще посчитать количество зарядов, на проводнике выбирают точку в любом удобном месте.

Через эту точку мысленно проводят плоскость, располагая ее перпендикулярно по отношению к проводнику. Так как эта плоскость в проводнике ограничивает собой площадь S, ее часто называют площадью поперечного сечения проводника.

Для вычисления силы тока, ведут подсчет зарядов, прошедших через это сечение.

Как рассчитать площадь сечения

Проводник будем считать круглой трубкой, по аналогии с трубой, по которой течет жидкость. Пользуясь этой аналогией, так же, примем, что внутри такой трубки будут двигаться заряды, они обозначены кружками на рисунке.{2} \right)\) – площадь круга;

\(\large \pi \approx 3,14\) – число Пи;

\(\large D \left(\text{м}\right)\) – диаметр круга;

\(\large R \left(\text{м}\right)\) – радиус круга;

Проводник может иметь не только цилиндрическую форму. Промышленность изготавливает металлические проводники, имеющие квадратное, прямоугольное, треугольное или какое-либо другое сечение. Понятно, что площади таких сечений нужно рассчитывать, пользуясь другими геометрическими формулами.

Сила тока по определению

Силу тока (ток) обозначают большой латинской буквой \(\large I\).

Постоянный ток можно рассматривать, как равномерное направленное движение заряженных частиц. Равномерное – значит, с одной и той же скоростью.

Если же ток изменяется, то будет изменяться и скорость движения зарядов.

Сила тока – это:

  1. физическая величина;
  2. отношение заряда, прошедшего через поперечное сечение проводника к длительности промежутка времени, в течение которого заряд проходил.

Ток равен заряду, прошедшему через поперечное сечение проводника за одну секунду.

Для постоянного тока используем формулу:

\[\large \boxed{ I = \frac{\Delta q}{\Delta t} }\]

\(\large I \left(A\right)\) – ток (сила тока) в Амперах;

\(\large \Delta q \left( \text{Кл}\right) \) – заряд в Кулонах, прошедший через поперечное сечение проводника;

\(\large \Delta t \left( c\right) \) – промежуток (кусочек) времени, в течение которого заряд прошел;

\[\large \boxed{ 1 A = \frac{1 \text{Кл}}{1 c} }\]

Если электрический ток не изменяется ни по величине, ни по направлению, то его называют постоянным.

Если хотя бы одна из характеристик изменяется, ток называют переменным. Он будет различным в разные моменты времени. Если задано уравнение, описывающее, как изменяется заряд, то для вычисления такого тока удобно пользоваться производной.

Исключаем путаницу с понятием силы

В физике исторически сложилось использование таких терминов, как

  • сила тока,
  • электродвижущая сила,
  • лошадиная сила.

Эти единицы измерения имеют в своем названии слово «сила». Из механики известно, что сила – величина векторная, измеряется в Ньютонах. Однако, пусть это не вводит вас в заблуждение.

Ни одна из описанных величин не измеряется в Ньютонах. Перечисленные величины имеют другие единицы измерения:

  • силу тока измеряют в Амперах,
  • электродвижущую силу – в Вольтах,
  • а лошадиная сила – это единица измерения мощности, ее можно перевести в Ватты в системе СИ.

Чтобы исключить путаницу, вместо термина «сила тока», можно употреблять слово «ток». Сравните выражения: «Силу тока измеряют в Амперах» и «ток измеряют в Амперах».

Как видно, вполне можно обойтись словом «ток», вместо «силы тока». Смысл от этого не изменится.

Что такое 1 Ампер в системе СИ

Сила тока в 1 Ампер была определена в системе СИ с помощью силы взаимного действия двух проводников с током.

Рассмотрим два тонких проводника (рис. 9). Каждый проводник имеет бесконечную длину.{-7} \) Ньютона, приходящейся на каждый метр их длины, то по каждому из них течет постоянный ток 1 Ампер.

Ампер – это основная единица в системе СИ. А заряд Кулон – величина, определяемая с помощью Ампера.

1 Кулон – это заряд, проходящий за 1 секунду через поперечное сечение проводника с током 1 Ампер.

Один Ампер – много это, или мало

1 Ампер это 1 Кулон деленный на 1 секунду. Для большинства бытовых электроприборов это достаточно большая сила тока.

Например, через энергосберегающие лампы протекают токи 0,04 — 0,08 Ампера.

Большой плоский телевизор от электроосветительной сети потребляет ток 0,2 Ампера.

Лампа накаливания –примерно 0,5 Ампера.

Как видно, большинство электроприборов потребляют токи менее одного Ампера.

Поэтому, для тока часто применяют дольные единицы измерения:

миллиамперы, микроамперы, и наноамперы:

1мА (миллиампер)= 10⁻³ А

1мкА (микроампер) = 10⁻⁶ А

1нА (наноампер) = 10⁻9 А

Ток зарядки аккумулятора мобильного телефона может достигать 2 Ампер.

А через электрический обогреватель, или электрочайник, протекает ток силой до 10 Ампер.

Примечание: Ток силой всего 0,05 А может привести к летальному исходу. Будьте осторожны с электричеством!

В то же время, используют и токи, превышающие сотни Ампер. Например, на промышленных электростанциях.

Для таких токов применяют кратные единицы: килоампер, мегаампер.

1КА (килоампер)= 10³ А

1МА (мегаампер) = 10⁶ А

Связь между силой тока и скоростью движения зарядов

Рассмотрим металлический проводник. Мысленно выделим в нем два сечения площадью \(\large S \) на некотором расстоянии \(\large \Delta x\) одно от другого. Сечения располагаются поперечно проводнику.

В металлах электрический ток создается электронами. Обозначим \(\large e_{0}\) заряд каждого электрона.

Рис. 10. Свободные заряды в объеме проводника

Заряды в проводнике, под действием электрического поля напряженностью \(\large \vec{E} \) будут двигаться сонаправленно, от сечения к сечению.

При этом, они будут проходить путь \(\large \Delta x\) между двумя сечениями.

Если ток постоянный, то скорость движения зарядов изменяться не будет.

В таком случае, расстояние \(\large \Delta x\) и скорость \(\large v\) движения электронов будут связаны формулой равномерного движения.

\[\large \Delta x = v \cdot \Delta t\]

\(\large \Delta x \left( \text{м}\right) \) – расстояние между двумя поперечными сечениями;

\(\large v \left( \frac{\text{м}}{c}\right) \) – скорость, с которой сонаправленно движутся заряды в проводнике; Эта скорость значительно меньше скорости теплового движения.

\(\large \Delta t \left( c \right) \) – интервал времени, за который пройдено расстояние \(\large \Delta x\) между двумя поперечными сечениями;

Выразим из этой формулы время движения:

\[\large \Delta t = \frac {\Delta x}{v} \]

Это выражение нам понадобится далее.

Сечения \(\large S \)  и расстояние между ними \(\large \Delta x\) образуют в проводнике цилиндрический объем:

\[\large V = S \cdot \Delta x\]

\(\large V \left( \text{м}^{3}\right) \) – объем цилиндра;

В этом объеме содержится определенное количество электронов.{3}}\right) \) – концентрация зарядов в объеме;

Найдем общий заряд всех заряженных частиц, расположенных в объеме \(\large V\) между двумя поперечными сечениями:

\[\large \Delta q = e_{0} \cdot N\]

Умножим правую часть уравнения на единицу, которую представим в виде дроби \(\displaystyle \frac{V}{V}\), в которой \(\large V\) – это рассматриваемый объем. Тогда полный заряд можно записать в таком виде:

\[\large \Delta q = e_{0} \cdot N \cdot 1 = e_{0} \cdot N \cdot \frac{V}{V}\]

Числитель V дроби и количество N частиц поменяем местами.

\[\large \Delta q = e_{0} \cdot V \cdot \frac{N}{V}\]

Подставим в эту формулу выражение для объема:

\[\large \Delta q = e_{0} \cdot S \cdot \Delta x \cdot \frac{N}{V}\]

Дробь в правой части заменим символом «n» концентрации:

\[\large \Delta q = e_{0} \cdot S \cdot \Delta x \cdot n\]

Средняя скорость совместного направленного движения зарядов \(\large v\).

Применим определение силы тока:

\[\large I = \frac {\Delta q}{\Delta t} \]

Подставим в это выражение формулу для общего заряда, прошедшего через сечение проводника:

\[\large I = \frac {\Delta q}{\Delta t} =  \frac {e_{0} \cdot S \cdot \Delta x \cdot n}{\Delta t} \]

Выражение для удобства можно переписать так:

\[\large I = e_{0} \cdot S \cdot \Delta x \cdot n\cdot \frac {1}{\Delta t} \]

Мы заранее выразили время \(\large \Delta t \):

\[\large \Delta t = \frac {\Delta x}{v} \]

Найдем для него обратную величину:

\[\large \frac {1}{\Delta t} = \frac {v}{\Delta x} \]

Подставим ее в формулу для тока:

\[\large I = e_{0} \cdot S \cdot \Delta x \cdot n \cdot \frac {v}{\Delta x}\]

Расстояние \(\Delta x\) находится в числителе и в знаменателе, оно сократится. Окончательно получим выражение для связи между силой тока и скоростью движения зарядов:

\[\large \boxed{I = e_{0} \cdot S \cdot n \cdot {v}}\]

Теперь можно утверждать, что

  • чем больше зарядов помещаются в объеме,
  • чем быстрее они сонаправленно двигаются
  • и, чем толще проводник (чем больше площадь поперечного сечения),

тем больше ток.

Расчет скорости направленного движения электронов

Для этого можно использовать полученную формулу:

\[\large I = e_{0} \cdot S \cdot n \cdot {v}\]

Из нее можно выразить скорость:

\[\large \boxed{\frac{I}{e_{0} \cdot S \cdot n} = v}\]

Чтобы найти скорость, с которой электроны движутся в проводнике, нужно: ток (I) разделить на заряд (е) электрона, концентрацию (n) электронов и площади сечения проводника (S).

Большинство соединительных проводников изготавливают из меди, или алюминия. Выберем медный проводник, имеющий цилиндрическую форму.{8} \left( \frac{\text{м}}{c} \right)\]

А скорость направленного движения зарядов значительно меньше — менее 0,1 мм в секунду.

Рис. 12. Заряды одновременно участвуют в двух движениях

Примечание: В качестве скорости направленного движения свободных зарядов, выбирают среднее значение скорости, с которой перемещаются заряды во время протекания тока. Ее, так же, называют скоростью дрейфа.

В то же время, при комнатной температуре скорость беспорядочного теплового движения электронов немногим более 100 километров в секунду.

То есть, заряды быстро движутся хаотично, но при этом, они согласованно и достаточно медленно передвигаются в определенном направлении.

Такое движение можно сравнить с движением потока муравьев на лесной тропе. Каждый муравей в потоке движется хаотично. Но при этом, весь поток движется согласованно в выбранную сторону.

Рис. 13. Движение муравьев и движение зарядов во время протекания электрического тока можно сравнить

Пользуясь аналогией из окружающей природы, движение заряженных частиц во время протекания электрического тока можно сравнить с движением муравьев.

Каждая частица движется хаотически под действием температуры и одновременно с этим, все частицы смещаются в одном направлении в общем потоке под действием электрического поля.

Условия существования постоянного тока

Напомню, что ток называют постоянным, если его сила не изменяется со временем.

Для обозначения постоянного тока математики используют такую сокращенную запись:

\[\large \boxed{ I = const }\]

Чтобы ток мог существовать, нужно, чтобы выполнялись несколько условий.

Рис. 14. Чтобы ток существовал, нужно, чтобы выполнялись такие условия

Нужно, чтобы между телами, заряженными противоположно, непрерывно существовало электрическое поле. Так же, в цепи должны присутствовать свободные носители заряда. А сама электрическая цепь должна быть замкнутой.

Рассмотрим эти условия подробнее.

Создаем кратковременный ток и выясняем условия его существования

Можно создать электрический ток с помощью двух заряженных противоположно тел.

Ток – это движение зарядов. Поэтому, нужно обеспечить возможность зарядам двигаться. То есть, нужно создать между телами дорожку, по которой заряды начнут перемещаться из одного места пространства в другое.

Продемонстрировать возникновение тока на небольшой промежуток времени можно с помощью двух электрометров, заряженных противоположно.

Попробуем для начала соединить два заряженных тела куском диэлектрика (рис. 15).

Рис. 15. Если диэлектриком соединить два заряженных тела, электрический ток не возникает

Как видно, после соединения заряд каждого из электрометров не изменился.

Это значит, что ток не возник. Дело в том, что в диэлектрике все электроны связаны со своими атомами и свободных электронов нет.

Именно свободные заряды будут передвигаться и их согласованное направленное движение мы назовем электрическим током.

Поэтому, одним из условий существования тока будет наличие свободных зарядов. То есть, наличие проводника, содержащего такие заряды.

Условие 1. Чтобы ток существовал, требуется наличие свободных зарядов.

Однако, только лишь наличия проводника недостаточно. Действительно, в проводнике присутствуют свободные заряды. Но для того, чтобы эти заряды начали совместное движение в определенную сторону, нужно, чтобы на них подействовала сила, которая будет их передвигать в этом направлении.

Сила будет действовать на заряженную частицу, если ее поместить в электрическом поле.

Электрическое поле существует в пространстве вокруг заряженных тел.

Если соединить проводником два тела, имеющие противоположные заряды, то на свободные частицы в проводнике будет действовать электрическое поле. Это поле подхватит заставит двигаться электроны в определенном направлении.

Поэтому, еще одно условие для возникновения тока – это электрическое поле.

Рис. 16. После соединения проводником, заряженные противоположными зарядами электрометры разрядились

Условие 2. Чтобы ток существовал, требуется наличие электрического поля.

Рис. 17. Электроны двигаются против направления электрического поля

Ток течет в направлении движения положительных зарядов.

Соединив два заряженных металлических тела проводником, мы получим ток лишь на короткий промежуток времени. Это время будет составлять доли секунды.

Кроме того, в начальный момент времени сила тока будет самой большой. А далее будет убывать по мере того, как тела будут разряжаться и их потенциалы (ссылка) будут выравниваться.

Мы же хотим, чтобы ток протекал постоянно, или, по крайней мере, достаточно длительный промежуток времени, выбранный по нашему усмотрению. И чтобы во время протекания тока его сила не изменялась.

Как этого добиться? Мы вплотную приблизились к третьему условию существования постоянного электрического тока.

Как создать длительный ток и что для этого необходимо

Положительный заряд – это недостаток электронов, а отрицательный – это их избыток. В момент соединения тел проводником, отрицательные электроны устремились к положительно заряженному телу.

А в конце ток прекратился потому, что заряды тел скомпенсировались и тела превратились в электрически нейтральные. Нам известно, что нейтральные тела электрическое поле не создают.

Значит, ток существует до тех пор, пока существует электрическое поле. Поэтому, нужно каким-либо образом поддерживать электрическое поле. А для этого нужно, чтобы одно из тел обладало избыточным отрицательным зарядом. То есть, нужно поддерживать на одном из тел отрицательный, а на другом – положительный заряд. Пока заряды тел будут поддерживаться, ток будет существовать.

Чтобы на теле с положительным зарядом поддерживать этот заряд, нужно убирать с этого тела прибежавшие туда электроны и отправлять их обратно на отрицательно заряженное тело.

Такая схема по своему устройству напоминает фонтан, в котором насос поддерживает разность давлений. В нагнетающей воду трубе давление больше, чем в трубе, через которую вода поступает обратно в насос.

Рис. 18. Поток воды циркулирует благодаря насосу, поддерживающему разность давлений

Именно благодаря этой разности, из одной трубы вода выплескивается вверх, а собранная в чашу вода попадает обратно в насос. При этом, по контуру циркулирует одно и то же количество воды, то есть, водяной контур замкнут. А ток воды в этом контуре поддерживается специальным устройством – насосом. Он совершает работу против силы тяжести.

Рис. 19. Водяной насос в фонтане совершает работу против силы тяжести

Сторонние силы — что это такое

Подобно своеобразному насосу устроен источник тока. Внутри источника действуют сторонние силы. Они возвращают электроны на «-» контакт.

На заряды в электрическом поле будет действовать сила. Она называется силой Кулона и имеет электрическую природу. Электроны будут притягиваться к телу, имеющему положительный заряд.

Сила Кулона будет мешать возвращать электроны на отрицательное тело. Подобно силе тяжести, которая мешает воде в фонтане двигаться вверх.

Чтобы вернуть электроны на отрицательно («-») заряженное тело, нужно совершить работу против силы Кулона. Значит, должна присутствовать какая-то внешняя сила, возвращающая электроны на отрицательно («-») заряженное тело. Эта сила имеет неэлектрическую природу, она называется сторонней силой.

Рис. 20. Источник тока совершает работу против электрической силы Кулона

Теперь можно ответить на вопрос: Что такое источник тока?

Источник тока — это устройство, внутри которого сторонние силы перемещают заряды против сил Кулона. Сила Кулона – это сила, с которой электростатическое поле действует на заряд.

Во время существования электрического тока сами электроны не расходуются. Они, как вода в фонтане, циркулируют по замкнутой траектории.

Условие 3. Чтобы ток существовал длительно, электрическое поле нужно долговременно поддерживать.

Чтобы ток существовал постоянно, нужно, чтобы между заряженными противоположно телами электрическое поле существовало непрерывно.

Примечание: В качестве заряженных противоположно тел можно рассматривать контакты источника тока.

Для этого электроны нужно пропустить по замкнутому контуру, т. е. непрерывной электрической цепи. Поэтому, еще одно условие существования постоянного тока – это замкнутая электрическая цепь. Как только замыкается цепь, в направленное движение приходят все заряженные частицы, находящиеся в этой цепи.

Условие 4. Чтобы ток существовал, требуется, чтобы электрическая цепь была замкнутой.

Рис. 21. Электрический ток источник может создать только в замкнутой цепи

В такой цепи заряды циркулируют по замкнутой траектории. То есть, заряд, вышедший из источника и совершивший полный оборот, попадет обратно в источник тока. Там он будет подхвачен сторонними силами и через противоположный вывод источника тока попадает обратно в цепь. Затем, будет двигаться далее и, совершит следующий круг. Поэтому, во время протекания электрического тока сами заряды не расходуются.

Во время протекания электрического тока заряды не расходуются. То есть, по замкнутой цепи двигаются одни и те же заряды. Совершив круг, они попадают в источник и, выходя из противоположного его вывода направляются обратно в цепь.

Нам известно, если на заряд действует сила и, под действием этой силы заряд перемещается, то эта сила совершает работу.

Это значит, что сторонние силы в источнике совершают работу. Подробнее о работе сторонних сил (ссылка).

 

По определению сила тока равна. Как измерять силу тока в электрической цепи

Содержание:

Движение заряженных частиц в проводнике в электротехнике называется электрическим током. Электроток не характеризуется только прошедшим через проводник значением количества электрической энергии, так как за 60 минут через него может пройти электричество равное 1 Кулону, но и такое же количество электричества можно пропустить через проводник за одну секунду.

Что такое сила тока

Когда рассматривается количество электричества, протекающее через проводник за разные интервалы времени, понятно, что за меньший промежуток времени ток течет интенсивней, поэтому в характеристику электротока вводится еще одно определение — это сила тока, которая характеризуется протекающим в проводнике током за секунду времени. Единицей измерения величины силы проходящего тока в электротехнике принят ампер.

Иными словами, сила электрического тока в проводнике — это количество электричества, которое прошло через его сечение за секунду времени, маркировка литерой I. Силу тока измеряют в амперах — это единица измерения, которая равняется силе неизменяющегося тока, проходящего по бесконечным параллельным проводам с наименьшим круговым сечением, удаленным друг от друга на 100 см и расположенным в вакууме, который вызывает взаимодействие на метре длины проводника силой = 2*10 минус 7 степени Ньютона на каждые 100 см длины.

Специалисты часто определяют величину проходящего тока, на Украине (сила струму) она равна 1 амперу, когда через сечение проводника проходит каждую секунду 1 кулон электричества.

В электротехнике можно увидеть частое применение других величин в определении значения силы проходящего тока: 1 миллиампер, который равен единица/ Ампер, 10 в минус третьей степени Ампер, один микроампер — это десять в минус шестой степени Ампер.

Зная количество электричества, прошедшее через проводник за определенный промежуток времени, можно вычислить силу тока (как говорят на Украине — силу струму) по формуле:

Когда электрическая цепь замкнута и не имеет ответвлений, тогда в каждом месте ее поперечного сечения протекает за секунду одинаковое количество электричества. Теоретически это объясняется невозможностью накапливания электрических зарядов в каком либо месте цепи, по этой причине сила тока везде одинакова.

Данное правило справедливо и в сложных цепях, когда есть ответвления, но относится к некоторым участкам сложной цепи, которые можно рассматривать в виде простой электроцепи.

Как измеряется сила тока

Величину силы тока измеряют прибором, который называется амперметр, а также для небольших значений — миллиамперметр и микроамперметр, который можно увидеть на фото внизу:

Среди людей бытует мнение, что когда измеряется сила тока в проводнике до нагрузки (потребителя), то значение будет выше, чем после нее. Это ошибочное мнение, основанное на том, что якобы какое-то значение силы будет расходоваться на то, чтобы привести потребитель в действие. Электроток в проводнике — это процесс электромагнитный, в котором участвуют заряженные электроны, они направленно двигаются, но энергию передают не электроны, а электромагнитное поле, которое окружает проводник.

Количество электронов, вышедших из начала цепи, будет равно количеству электронов и после потребителя в конце цепи, они не могут быть израсходованы.

Какие проводники бывают? Специалисты дают определение понятию «проводник» — это материал, в котором частицы, имеющие заряд, могут перемещаться свободно. Такие свойства на практике имеют почти все металлы, кислота и солевой раствор. А материал или вещество, в котором движение заряженных частиц затруднено или вообще невозможно, называются изоляторами (диэлектриками). Часто встречающиеся материалы-диэлектрики — это кварц или эбонит, искусственный изолятор.

Вывод

На практике современное оборудование работает с большими величинами тока, до сотни, а то и тысячи ампер, а также и с малыми значениями. Примером в повседневной жизни величины тока в разных приборах может быть электрическая плита, где она достигает значения в 5 А, а простая лампа накаливания может иметь величину 0,4 А, в фотоэлементе величина проходящего тока измеряется в микроамперах. В линиях городского общественного транспорта (троллейбус, трамвай) значение проходящего тока достигает 1000 А.

Если изолированный проводник поместить в электрическое поле \(\overrightarrow{E} \), то на свободные заряды \(q\) в проводнике будет действовать сила \(\overrightarrow{F} = q\overrightarrow{E}\) В результате в проводнике возникает кратковременное перемещение свободных зарядов. Этот процесс закончится тогда, когда собственное электрическое поле зарядов, возникших на поверхности проводника, скомпенсирует полностью внешнее поле. Результирующее электростатическое поле внутри проводника будет равно нулю.

Однако, в проводниках при определенных условиях может возникнуть непрерывное упорядоченное движение свободных носителей электрического заряда.

Направленное движение заряженных частиц называется электрическим током.

За направление электрического тока принято направление движения положительных свободных зарядов. Для существования электрического тока в проводнике необходимо создать в нем электрическое поле.

Количественной мерой электрического тока служит сила тока \(I\) — скалярная физическая величина, равная отношению заряда \(\Delta q\), переносимого через поперечное сечение проводника (рис. 1.8.1) за интервал времени \(\Delta t\), к этому интервалу времени:

$$I = \frac{\Delta q}{\Delta t} $$

Если сила тока и его направление не изменяются со временем, то такой ток называется постоянным .

В Международной системе единиц СИ сила тока измеряется в Амперах (А). Единица измерения тока 1 А устанавливается по магнитному взаимодействию двух параллельных проводников с током.

Постоянный электрический ток может быть создан только в замкнутой цепи , в которой свободные носители заряда циркулируют по замкнутым траекториям. Электрическое поле в разных точках такой цепи неизменно во времени. Следовательно, электрическое поле в цепи постоянного тока имеет характер замороженного электростатического поля. Но при перемещении электрического заряда в электростатическом поле по замкнутой траектории, работа электрических сил равна нулю. Поэтому для существования постоянного тока необходимо наличие в электрической цепи устройства, способного создавать и поддерживать разности потенциалов на участках цепи за счет работы сил неэлектростатического происхождения . Такие устройства называются источниками постоянного тока . Силы неэлектростатического происхождения, действующие на свободные носители заряда со стороны источников тока, называются сторонними силами .

Природа сторонних сил может быть различной. В гальванических элементах или аккумуляторах они возникают в результате электрохимических процессов, в генераторах постоянного тока сторонние силы возникают при движении проводников в магнитном поле. Источник тока в электрической цепи играет ту же роль, что и насос, который необходим для перекачивания жидкости в замкнутой гидравлической системе. Под действием сторонних сил электрические заряды движутся внутри источника тока против сил электростатического поля, благодаря чему в замкнутой цепи может поддерживаться постоянный электрический ток.

При перемещении электрических зарядов по цепи постоянного тока сторонние силы, действующие внутри источников тока, совершают работу.

Физическая величина, равная отношению работы \(A_{ст}\) сторонних сил при перемещении заряда \(q\) от отрицательного полюса источника тока к положительному к величине этого заряда, называется электродвижущей силой источника (ЭДС):

$$ЭДС=\varepsilon=\frac{A_{ст}}{q}. $$

Таким образом, ЭДС определяется работой, совершаемой сторонними силами при перемещении единичного положительного заряда. Электродвижущая сила, как и разность потенциалов, измеряется в Вольтах (В).

При перемещении единичного положительного заряда по замкнутой цепи постоянного тока работа сторонних сил равна сумме ЭДС, действующих в этой цепи, а работа электростатического поля равна нулю.

Цепь постоянного тока можно разбить на отдельные участки. Те участки, на которых не действуют сторонние силы (т. е. участки, не содержащие источников тока), называются однородными . Участки, включающие источники тока, называются неоднородными .

При перемещении единичного положительного заряда по некоторому участку цепи работу совершают как электростатические (кулоновские), так и сторонние силы. Работа электростатических сил равна разности потенциалов \(\Delta \phi_{12} = \phi_{1} — \phi_{2}\) между начальной (1) и конечной (2) точками неоднородного участка. Работа сторонних сил равна по определению электродвижущей силе \(\mathcal{E}\), действующей на данном участке. Поэтому полная работа равна

$$U_{12} = \phi_{1} — \phi_{2} + \mathcal{E}$$

Величину U 12 принято называть напряжением на участке цепи 1-2. В случае однородного участка напряжение равно разности потенциалов:

$$U_{12} = \phi_{1} — \phi_{2}$$

Немецкий физик Г. Ом в 1826 году экспериментально установил, что сила тока \(I\), текущего по однородному металлическому проводнику (т. е. проводнику, в котором не действуют сторонние силы), пропорциональна напряжению \(U\) на концах проводника:

$$I = \frac{1}{R} U; \: U = IR$$

где \(R\) = const.

Величину R принято называть электрическим сопротивлением . Проводник, обладающий электрическим сопротивлением, называется резистором . Данное соотношение выражает закон Ома для однородного участка цепи: сила тока в проводнике прямо пропорциональна приложенному напряжению и обратно пропорциональна сопротивлению проводника.

В СИ единицей электрического сопротивления проводников служит Ом (Ом). Сопротивлением в 1 Ом обладает такой участок цепи, в котором при напряжении 1 В возникает ток силой 1 А.

Проводники, подчиняющиеся закону Ома, называются линейными . Графическая зависимость силы тока \(I\) от напряжения \(U\) (такие графики называются вольт-амперными характеристиками , сокращенно ВАХ) изображается прямой линией, проходящей через начало координат. Следует отметить, что существует много материалов и устройств, не подчиняющихся закону Ома, например, полупроводниковый диод или газоразрядная лампа. Даже у металлических проводников при токах достаточно большой силы наблюдается отклонение от линейного закона Ома, так как электрическое сопротивление металлических проводников растет с ростом температуры.

Для участка цепи, содержащего ЭДС, закон Ома записывается в следующей форме:

$$IR = U_{12} = \phi_{1} — \phi_{2} + \mathcal{E} = \Delta \phi_{12} + \mathcal{E}$$
$$\color{blue}{I = \frac{U}{R}}$$

Это соотношение принято называть обобщенным законом Ома или законом Ома для неоднородного участка цепи .

На рис. 1.8.2 изображена замкнутая цепь постоянного тока. Участок цепи (cd ) является однородным.

Рисунок 1.8.2.

Цепь постоянного тока

По закону Ома

$$IR = \Delta\phi_{cd}$$

Участок (ab ) содержит источник тока с ЭДС, равной \(\mathcal{E}\).

По закону Ома для неоднородного участка,

$$Ir = \Delta \phi_{ab} + \mathcal{E}$$

Сложив оба равенства, получим:

$$I(R+r) = \Delta\phi_{cd} + \Delta \phi_{ab} + \mathcal{E}$$

Но \(\Delta\phi_{cd} = \Delta \phi_{ba} = -\Delta \phi_{ab}\).

$$\color{blue}{I=\frac{\mathcal{E}}{R + r}}$$

Эта формула выражает закон Ома для полной цепи : сила тока в полной цепи равна электродвижущей силе источника, деленной на сумму сопротивлений однородного и неоднородного участков цепи (внутреннего сопротивления источника).

Сопротивление r неоднородного участка на рис. 1.8.2 можно рассматривать как внутреннее сопротивление источника тока . В этом случае участок (ab ) на рис. 1.8.2 является внутренним участком источника. Если точки a и b замкнуть проводником, сопротивление которого мало по сравнению с внутренним сопротивлением источника (\(R\ \ll r\)), тогда в цепи потечет ток короткого замыкания

$$I_{кз}=\frac{\mathcal{E}}{r}$$

Сила тока короткого замыкания — максимальная сила тока, которую можно получить от данного источника с электродвижущей силой \(\mathcal{E}\) и внутренним сопротивлением \(r\). У источников с малым внутренним сопротивлением ток короткого замыкания может быть очень велик и вызывать разрушение электрической цепи или источника. Например, у свинцовых аккумуляторов, используемых в автомобилях, сила тока короткого замыкания может составлять несколько сотен ампер. Особенно опасны короткие замыкания в осветительных сетях, питаемых от подстанций (тысячи ампер). Чтобы избежать разрушительного действия таких больших токов, в цепь включаются предохранители или специальные автоматы защиты сетей.

В ряде случаев для предотвращения опасных значений силы тока короткого замыкания к источнику последовательно подсоединяется некоторое внешнее сопротивление. Тогда сопротивление r равно сумме внутреннего сопротивления источника и внешнего сопротивления, и при коротком замыкании сила тока не окажется чрезмерно большой.

Если внешняя цепь разомкнута, то \(\Delta \phi_{ba} = -\Delta \phi_{ab} = \mathcal{E}\), т. е. разность потенциалов на полюсах разомкнутой батареи равна ее ЭДС.

Если внешнее нагрузочное сопротивление R включено и через батарею протекает ток I , разность потенциалов на ее полюсах становится равной

$$\Delta \phi_{ba} = \mathcal{E} — Ir$$

На рис. 1.8.3 дано схематическое изображение источника постоянного тока с ЭДС равной \(\mathcal{E}\) и внутренним сопротивлением r в трех режимах: «холостой ход», работа на нагрузку и режим короткого замыкания (к. з.). Указаны напряженность \(\overrightarrow{E}\) электрического поля внутри батареи и силы, действующие на положительные заряды:\(\overrightarrow{F}_{э}\) — электрическая сила и \(\overrightarrow{F}_{ст}\) — сторонняя сила. В режиме короткого замыкания электрическое поле внутри батареи исчезает.

Для измерения напряжений и токов в электрических цепях постоянного тока используются специальные приборы — вольтметры и амперметры .

Вольтметр предназначен для измерения разности потенциалов, приложенной к его клеммам. Он подключается параллельно участку цепи, на котором производится измерение разности потенциалов. Любой вольтметр обладает некоторым внутренним сопротивлением \(R_{В}\). Для того, чтобы вольтметр не вносил заметного перераспределения токов при подключении к измеряемой цепи, его внутреннее сопротивление должно быть велико по сравнению с сопротивлением того участка цепи, к которому он подключен. Для цепи, изображенной на рис. 1.8.4, это условие записывается в виде:

$$R_{В} \gg R_{1}$$

Это условие означает, что ток \(I_{В} = \Delta \phi_{cd} / R_{В}\), протекающий через вольтметр, много меньше тока \(I = \Delta \phi_{cd} / R_{1}\), который протекает по тестируемому участку цепи.

Поскольку внутри вольтметра не действуют сторонние силы, разность потенциалов на его клеммах совпадает по определению с напряжением. Поэтому можно говорить, что вольтметр измеряет напряжение.

Амперметр предназначен для измерения силы тока в цепи. Амперметр включается последовательно в разрыв электрической цепи, чтобы через него проходил весь измеряемый ток. Амперметр также обладает некоторым внутренним сопротивлением \(R_{А}\). В отличие от вольтметра, внутреннее сопротивление амперметра должно быть достаточно малым по сравнению с полным сопротивлением всей цепи. Для цепи на рис. 1.8.4 сопротивление амперметра должно удовлетворять условию

$$R_{А} \ll (r + R_{1} + R{2})$$

чтобы при включении амперметра ток в цепи не изменялся.

Измерительные приборы — вольтметры и амперметры — бывают двух видов: стрелочные (аналоговые) и цифровые. Цифровые электроизмерительные приборы представляют собой сложные электронные устройства. Обычно цифровые приборы обеспечивают более высокую точность измерений.

«Физика — 10 класс»

Электрический ток — направленное движение заряженных частиц. Благодаря электрическому току освещаются квартиры, приводятся в движение станки, нагреваются конфорки на электроплитах, работает радиоприемник и т. д.

Рассмотрим наиболее простой случай направленного движения заряженных частиц — постоянный ток.

Какой электрический заряд называется элементарным?
Чему равен элементарный электрический заряд?
Чем различаются заряды в проводнике и диэлектрике?

При движении заряженных частиц в проводнике происходит перенос электрического заряда из одной точки в другую. Однако если заряженные частицы совершают беспорядочное тепловое движение, как, например, свободные электроны в металле, то переноса заряда не происходит (рис. 15.1, а). Поперечное сечение проводника в среднем пересекает одинаковое число электронов в двух противоположных направлениях. Электрический заряд переносится через поперечное сечение проводника лишь в том случае, если наряду с беспорядочным движением электроны участвуют в направленном движении (рис. 15.1, б). В этом случае говорят, что по проводнику идёт электрический ток .

Электрическим током называют упорядоченное (направленное) движение заряженных частиц.

Электрический ток имеет определённое направление.

За направление тока принимают направление движения положительно заряженных частиц.

Если перемещать нейтральное в целом тело, то, несмотря на упорядоченное движение огромного числа электронов и атомных ядер, электрический ток не возникнет. Полный заряд, переносимый через любое сечение, будет при этом равным нулю, так как заряды разных знаков перемещаются с одинаковой средней скоростью.

Направление тока совпадает с направлением вектора напряжённости электрического поля. Если ток образован движением отрицательно заряженных частиц, то направление тока считают противоположным направлению движения частиц.

Выбор направления тока не очень удачен, так как в большинстве случаев ток представляет собой упорядоченное движение электронов — отрицательно заряженных частиц. Выбор направления тока был сделан в то время, когда о свободных электронах в металлах ещё ничего не знали.

Действие тока.

Движение частиц в проводнике мы непосредственно не видим. О наличии электрического тока приходится судить по тем действиям или явлениям, которые его сопровождают.

Во-первых, проводник, по которому идёт ток, нагревается.

Во-вторых, электрический ток может изменять химический состав проводника: например, выделять его химические составные части (медь из раствора медного купороса и т. д.).

В-третьих, ток оказывает силовое воздействие на соседние токи и намагниченные тела. Это действие тока называется магнитным .

Так, магнитная стрелка вблизи проводника с током поворачивается. Магнитное действие тока в отличие от химического и теплового является основным, так как проявляется у всех без исключения проводников. Химическое действие тока наблюдается лишь у растворов и расплавов электролитов, а нагревание отсутствует у сверхпроводников.

В лампочке накаливания вследствие прохождения электрического тока излучается видимый свет, а электродвигатель совершает механическую работу.

Сила тока.

Если в цепи идёт электрический ток, то это означает, что через поперечное сечение проводника всё время переносится электрический заряд.

Заряд, перенесённый в единицу времени, служит основной количественной характеристикой тока, называемой силой тока .

Если через поперечное сечение проводника за время Δt переносится заряд Δq, то среднее значение силы тока равно

Средняя сила тока равна отношению заряда Δq прошедшего через поперечное сечение проводника за промежуток времени Δt, к этому промежутку времени.

Если сила тока со временем не меняется, то ток называют постоянным .

Сила переменного тока в данный момент времени определяется также по формуле (15.1), но промежуток времени Δt в таком случае должен быть очень мал.

Сила тока, подобно заряду, — величина скалярная. Она может быть как положительной , так и отрицательной . Знак силы тока зависит от того, какое из направлений обхода контура принять за положительное. Сила тока I > 0, если направление тока совпадает с условно выбранным положительным направлением вдоль проводника. В противном случае I

Связь силы тока со скоростью направленного движения частиц.

Пусть цилиндрический проводник (рис. 15.2) имеет поперечное сечение площадью S.

За положительное направление тока в проводнике примем направление слева направо. Заряд каждой частицы будем считать равным q 0 . В объёме проводника, ограниченном поперечными сечениями 1 и 2 с расстоянием Δl между ними, содержится nSΔl частиц, где n — концентрация частиц (носителей тока). Их общий заряд в выбранном объёме q = q 0 nSΔl. Если частицы движутся слева направо со средней скоростью υ, то за время все частицы, заключенные в рассматриваемом объёме, пройдут через поперечное сечение 2. Поэтому сила тока равна:

В СИ единицей силы тока является ампер (А).

Эта единица установлена на основе магнитного взаимодействия токов.

Измеряют силу тока амперметрами . Принцип устройства этих приборов основан на магнитном действии тока.

Скорость упорядоченного движения электронов в проводнике.

Найдём скорость упорядоченного перемещения электронов в металлическом проводнике. Согласно формуле (15.2) где е — модуль заряда электрона.

Пусть, например, сила тока I = 1 А, а площадь поперечного сечения проводника S = 10 -6 м 2 . Модуль заряда электрона е = 1,6 10 -19 Кл. Число электронов в 1 м 3 меди равно числу атомов в этом объёме, так как один из валентных электронов каждого атома меди является свободным. Это число есть n ≈ 8,5 10 28 м -3 (это число можно определить, если решить задачу 6 из § 54). Следовательно,

Как видите, скорость упорядоченного перемещения электронов очень мала. Она во много раз меньше скорости теплового движения электронов в металле.

Условия, необходимые для существования электрического тока.

Для возникновения и существования постоянного электрического тока в веществе необходимо наличие свободных заряженных частиц.

Однако этого ещё недостаточно для возникновения тока.

Для создания и поддержания упорядоченного движения заряженных частиц необходима сила, действующая на них в определённом направлении.

Если эта сила перестанет действовать, то упорядоченное движение заряженных частиц прекратится из-за столкновений с ионами кристаллической решётки металлов или нейтральными молекулами электролитов и электроны будут двигаться беспорядочно.

На заряженные частицы, как мы знаем, действует электрическое поле с силой:

Обычно именно электрическое поле внутри проводника служит причиной, вызывающей и поддерживающей упорядоченное движение заряженных частиц.
Только в статическом случае, когда заряды покоятся, электрическое поле внутри проводника равно нулю.

Если внутри проводника имеется электрическое поле, то между концами проводника в соответствии с формулой (14.21) существует разность потенциалов. Как показал эксперимент, когда разность потенциалов не меняется во времени, в проводнике устанавливается постоянный электрический ток . Вдоль проводника потенциал уменьшается от максимального значения на одном конце проводника до минимального на другом, так как положительный заряд под действием сил поля перемещается в сторону убывания потенциала.

Сила тока

Характеристикой тока в цепи служит величина, называемая силой тока (I ). Сила тока – физическая величина, характеризующая скорость прохождения заряда через проводник и равная отношению заряда q , прошедшeгo через пoперeчное сечение проводника за промежуток времени t , к этому промежутку времени: I = q/t . Единица измерения силы тока – 1 ампер (1 А).

Определение единицы силы тока основано на магнитном действии тока, в частности на взаимодействии параллельных проводников, по которым идёт электрический ток. Такие проводники притягиваются, если ток по ним идёт в одном направлении, и отталкиваются, если направление тока в них противоположное.

За единицу силы тока принимают такую силу тока, при которой отрезки параллельных проводников длиной 1 м, находящиеся на расстоянии 1 м друг от друга, взаимодействуют с силой 2*10 -7 Н . Эта единица и называется ампером (1 А).

Зная формулу силы тока, можно получить единицу электрического заряда: 1 Кл = 1А * 1с.

Амперметр

Прибор, с помощью которого измеряют силу тока в цепи, называется амперметром . Его работа основана на магнитном действии тока. Основные части амперметра магнит и катушка . При прохождении по катушке электрического тока она в результате взаимодействия с магнитом, поворачивается и поворачивает соединённую с ней стрелку. Чем больше сила тока, проходящего через катушку, тем сильнее она взаимодействует с магнитом, тем больше угол поворота стрелки. Амперметр включается в цепь последовательно с тем прибором, силу тока в котором нужно измерить, и потому он имеет малое внутреннее сопротивление, которое практически не влияет на сопротивление цепи и на силу тока в цепи.

У клемм амперметра стоят знаки «+» и «-» , при включении амперметра в цепь клемма со знаком «+» присоединяется к положительному пoлюсу источника тока, а клемма со знаком «-» к отрицательному пoлюсу истoчникa тока.

Напряжение

Источник тока создаёт электрическое поле, которое приводит в движение электрические заряды. Характеристикой источника тока служит величина, называемая напряжением . Чем оно больше, тем сильнее созданное им поле. Напряжение характеризует работу, которую совершает электрическое поле по перемещению электрического заряда.

Напряжение (U ) — это физическая величина, равную отношению работы (А ) электрического поля по перемещению электрического заряда к заряду (q): U = A/q .

Возможно другое определение понятия напряжения. Если числитель и знаменатель в формуле напряжения умножить на время движения заряда (t ), то получим: U = At/qt . В числителе этой дроби стоит мощность тока (Р ), а в знаменателе — сила тока (I ). Получается формула: U = Р/I , т.е. напряжение — это физическая величина, равная отношению мощности электрического тока к силе тока в цепи.

Единица напряжения: [U ] = 1 Дж/1 Кл = 1 В (один вольт).

Вольтметр

Напряжение измеряют вольтметром. Он имеет такое же устройство, что и амперметр и такой же принцип действия, но он подключается параллельно тому участку цепи, напряжение на котором хотят. Внутреннее сопротивление вольтметра достаточно большое, соответственно проходящий через него ток мал по сравнению с током в цепи.

У клемм вольтметра стоят знаки «+» и «-» , при включении вольтметра в цепь клeмма со знаком «+» присоединяется к положительному полюсу источника тока, а клеммa со знаком «-» к отрицательному полюсу источника тока.

Формулы и определения.

1. Все проводники, используемые в электрических цепях , имеют условные обозначения для изображения на схемах и могут образовывать последовательные, параллельные и смешанные соединения.

2. Мощность тока – физическая величинa, хаpактеpизующая скорость превращения электрической энергии в другие её виды. Единица для измерения – 1 ватт (1 Вт). Измерительный прибор – ваттметр.

3. Сила тока – физическaя вeличина, характеpизующaя скоpость прохождения заряда через проводник и равная отношению заряда, пpoшедшего через попеpeчное сечение проводника, ко времени перемещения. Единица – 1 ампер (1 А). Измерительный прибор – амперметр (подключают последовательно).

4. Электрическое напряжение – физическaя вeличина, характеризующая электрическое поле, создающее ток, и равная отношению мощности тока к его силе. Единица – 1 вольт (1 В). Измерительный прибор – вольтметр (подключают параллельно)

Физика 8 класс. СИЛА ТОКА

Направленное движение заряженных частиц называется электрическим током.

Условия существования электрического тока в проводнике:
1. наличие свободных заряженных частиц (в металлическом проводнике — свободных электронов),
2. наличие электрического поля в проводнике
(электрическое поле в проводнике создается источниками тока.).

Электрический ток имеет направление.
За направление тока принимают направление движения положительно заряженных частиц.

Сила тока (I) — скалярная величина, равная отношению заряда q , прошедшего через поперечное сечение проводника, к промежутку времени t , в течение которого шел ток.

Сила тока показывает, какой заряд проходит через поперечное сечение проводника за единицу времени.

Единица измерения силы тока в системе СИ:
[I] = 1 A (ампер)

В 1948 г. было предложено в основу определения единицы силы тока положить явление взаимодействия двух поводников с током:

……………………

При прохождении тока по двум параллельным проводникам в одном направлении проводники притягиваются, а при прохождении тока по этим же проводникам в противоположных направлениях отталкиваются.

За единицу силы тока 1 А принимают силу тока, при которой два параллельных проводника длиной 1м, расположенные на растоянии 1м друг от друга, взаимодействуют с силой 0,0000002 Н.

АНДРЕ-МАРИ АМПЕР
(1775 — 1836)
— французский физик и математик

Ввел такие термины , как электростатика, электродинамика, соленоид, ЭДС, напряжение, гальванометр, электрический ток и т. д.;
— предположил, что, вероятно, возникнет новая наука об общих закономерностях процессов управления и предложил назвать ее «кибернетикой»;
— открыл явление механического взаимодействия проводников с током и правило определения направления тока;
— имеет труды во многих областях наук: ботанике, зоологии, химии, математике, кибернетике;

Его именем названа единица измерения силы тока — 1 Ампер.

ЭЛЕКТРИЧЕСКИЕ ТОКИ В ПРИРОДЕ.

Мы живем в океане электрических разрядов, создаваемых машинами, станками и людьми. Эти разряды — кратковременные электрические токи не так мощны, и мы их часто не замечаем. Но они все-таки существуют и могут принести немало вреда!

Что такое молния?

В результате движения и трения друг о друга воздушные слои в атмосфере электризуются. В облаках с течением времени скапливаются большие заряды . Они-то и являются причиной молний.
В момент, когда заряд облака станет большим, между его частями, имеющими противоположные по знаку заряды, проскакивает мощная электрическая искра – молния. Молния может образовываться между двумя соседними облаками и между облаком и поверхностью Земли. В этом случае под действием электрического поля отрицательного заряда нижней части облака поверхность Земли под облаком электризуется положительно. В результате молния ударяет в землю.
Природа молнии стала проясняться после исследований, проведенных в XVIII столетии русскими учеными М.В.Ломоносовым и Г.Рихманом и американским ученым Б.Франклином.

Обычно молнию рисуют бьющей сверху вниз . Между тем в действительности свечение
начинается снизу и только затем распространяется по вертикальному каналу.
Молния – точнее ее видимая фаза, оказывается, бьет снизу вверх!

ЗАГЛЯНИ НА КНИЖНУЮ ПОЛКУ!

А ЕСТЬ ЛИ ГРОМООТВОД У ТЕБЯ НА ДАЧЕ?

Одним из первых в мире громоотводов (молниеотводов) водрузил над крестом своего храма сельский священник из Моравии по имени Прокоп Дивиш, крестьянский сын, ученый и изобретатель.
Это было в июне 1754 года.
___

Первый в России молниеотвод появился в 1756 г. над Петропавловским собором в Петербурге.
Он был сооружен после того, как молния дважды ударила в шпиль собора и подожгла его.

Поделитесь статьей с друзьями:

Похожие статьи

Методы заряда автомобильных аккумуляторов при постоянном токе

Заряд автомобильных аккумуляторов можно производить от любого источника постоянного тока при условии, что его напряжение больше, чем напряжение заряжаемой батареи. Для заряда автомобильных аккумуляторов положительный вывод соединяют с положительным полюсом источника тока, а отрицательный — с отрицательным.

Методы заряда автомобильных аккумуляторов, заряд при постоянном токе и при постоянном напряжении, уравнительный, форсированный и модифицированный заряд.

Для любого момента заряда автомобильных аккумуляторов сила зарядного тока будет определяться следующим соотношением:

Iз = (Uзу-Uб) / R

где:

Uзу — напряжение зарядного устройства.
Uб — напряжение батареи.
R — суммарное сопротивление зарядной цепи.

Из этого выражения следует, что при равенстве напряжения зарядного устройства и батареи ток в цепи будет равен нулю. Если напряжение источника больше напряжения батареи, зарядный ток будет тем больше, чем выше напряжение зарядного устройства. В случае, когда напряжение зарядного устройства меньше напряжения батареи, происходит разряд батареи.

Напряжение автомобильных аккумуляторов при заряде изменяется в зависимости от температуры электролита и степени ее заряженности. Поэтому в процессе заряда необходима регулировка параметров зарядного устройства. Процесс может быть осуществлен различными режимами в зависимости от применяемых в зарядных устройствах систем регулирования напряжения или тока. В эксплуатации используются следующие основные системы заряда:

— При постоянном токе.
— При постоянном напряжении.

Заряд автомобильного аккумулятора при постоянном токе.

При таком заряде сила тока в течение всего времени заряда автомобильных аккумуляторов должна оставаться неизменной. Для этого в ходе заряда автомобильных аккумуляторов необходимо менять напряжение зарядного устройства или сопротивление цепи. Существует несколько способов регулирования силы зарядного тока.

Основные из них:

— Включение последовательно с батареей в зарядную цепь реостата.
— Применение регуляторов силы тока (например, тиристорных), которые периодическим включением и выключением дополнительного сопротивления в цепи заряда меняют силу тока таким образом, чтобы его среднее значение оставалось постоянным.
— Изменение напряжения источника тока ручным или автоматическим регулятором в соответствии с показаниями силы тока. Корректируя его до заданного постоянного значения.

Большинство выпрямительных устройств, предназначенных для заряда автомобильных аккумуляторов, питается от сети переменного тока. Имеет либо ступенчатую, либо плавную регулировку напряжения за счет изменения коэффициента трансформации. Поэтому в процессе заряда приходится периодически регулировать напряжение рукоятками соответствующего регулятора.

При включении реостата последовательно с батареей в зарядную цепь, параметры реостата выбираются такими, чтобы сечение обмоток реостата допускало без перегрева силу тока заданной величины. Пределы регулирования сопротивления реостата должны удовлетворять следующим соотношениям:

Rн = (Uзу — nUнз) / Iз
Rк = (Uзу — nUкз) / Iз

где:

Uнз — напряжение заряжаемых батарей в начале заряда.
Uкз — напряжение тех же батарей в конце заряда.
n — количество последовательно включенных батарей.

Во всех случаях напряжение зарядного устройства должно быть выше суммарного напряжения поставленных на заряд батарей. При заряде постоянным током исправных батарей общее время заряда может быть определено по их исходной степени заряженности. Зная емкость, которую необходимо сообщить при заряде батареи, можно определить время ее заряда.

При заряде полностью разряженных батарей в нормальный условиях (при комнатной температуре) процесс заряда в первый момент идет с максимальным коэффициентом использования тока. Повышение степени заряженности и возрастание поляризации приводят к увеличению суммарного внутреннего сопротивления батареи и увеличению потерь энергии на нагрев электролита, электродов и других элементов батареи.

Кроме того, на заключительной стадии заряда автомобильных аккумуляторов начинается вторичный процесс — электролиз воды, входящей в состав электролита. Выделяющийся при электролизе воды газ создает видимость кипения электролита. Это свидетельствует о завершении процесса заряда батареи. С целью снижения потерь энергии при заряде, уменьшения нагрева батареи и предохранения уровня электролита от чрезмерного снижения рекомендуется в конце процесса заряда снижать силу зарядного тока.

Для заряда при постоянном токе наиболее распространенным является режим, который состоит из двух ступеней.

— Первая ступень заряда производится при токе равном 0,1С20 (номинальной емкости) до тех пор, пока напряжение на батарее 12 В не достигнет 14,4 В (2,4 В на каждом аккумуляторе).
— Далее сила зарядного тока снижается вдвое до величины 0,05С20 (номинальной емкости).

Заряд при такой силе тока продолжается до постоянства напряжения и плотности электролита в аккумуляторах в течение 2 часов. При этом в конце заряда наблюдается бурное выделение газа, приводящее к кипению электролита.

При заряде батарей последнего поколения (без пробок для доливки воды) целесообразно при достижении напряжением заряда величины 15 В еще раз уменьшить ток заряда вдвое, т.е. до величины 0,025С20. Заряд таких батарей завершается при напряжении 16,2-16,4 В в зависимости от состава сплава решеток и чистоты электролита.

Пониженная сила тока в конце заряда автомобильных аккумуляторов позволяет:

— Уменьшить скорость газовыделения.
— Снизить влияние перегрева на последующую работоспособность и срок службы батареи.
— Обеспечить полноту заряда.

Учитывая, что при комнатной температуре напряжение на каждом аккумуляторе батареи составляет в конце заряда 2,6-2,7 В, расчетное сопротивление реостата должно выбираться из условия:

R = [Uзу — (2,6 / 2,7)] n/0,5 Iз

где:

n — количество аккумуляторов в заряжаемой батарее.
0,5 — корректирующий коэффициент, обеспечивающий возможность снижения зарядного тока при повышении температуры электролита.

Уравнительный заряд.

Такой заряд производится при постоянной силе тока менее 0,1 от номинальной емкости в течение несколько большего времени, чем обычно. Его цель — обеспечить полное восстановление активных масс во всех электродах всех аккумуляторов батареи.

Уравнительный заряд нейтрализует воздействие глубоких разрядов и рекомендуется как мера, устраняющая нарастающую сульфатацию электродов. Заряд продолжается до тех пор, пока во всех аккумуляторах батареи не будет наблюдаться постоянство плотности электролита и напряжения в течение трех часов. При необходимости в короткое время восстановить работоспособность глубоко разряженной аккумуляторной батареи, применяют так называемый форсированный заряд.

Форсированный заряд.

Такой заряд может производиться токами величиной до 70 % от номинальной емкости, но в течение более короткого времени. Время заряда тем меньше, чем больше величина зарядного тока. Практически при заряде током 0,7С20 продолжительность заряда автомобильных аккумуляторов не должна быть более 30 минут. При 0,5С20 — 45 минут, а при 0.ЗС20 — 90 минут. В процессе форсированного заряда необходимо контролировать температуру электролита, и при достижении 45 °С прерывать заряд.

Следует отметить, что применение форсированного заряда должно быть исключением. Так как его систематическое многократное повторение для одной и той же батареи заметно сокращает срок ее службы.

Заряд автомобильного аккумулятора при постоянном напряжении.

При этом способе в течение всего времени заряда напряжение зарядного устройства поддерживается постоянным. Зарядный ток убывает в процессе заряда по причине увеличения внутреннего сопротивления батареи.

В первый момент после включения сила зарядного тока определяется следующими факторами:

— Выходным напряжением источника питания.
— Степенью заряженности батареи.
— Количеством последовательно включенных батарей.
— Температурой электролита батарей.

Сила зарядного тока в начальный момент заряда может достигать (1,0-1,5)С20. Для исправных, но разряженных батарей такие токи не принесут вредных последствий. Несмотря на большие токи в начальный момент зарядного процесса общая продолжительность полного заряда батарей примерно соответствует режиму при постоянстве тока.

Дело в том, что заключительный этап заряда при постоянстве напряжения происходит при достаточно малой силе тока. Тем не менее, заряд по такой методике в ряде случаев предпочтителен. Так как он обеспечивает более быстрое доведение батареи до состояния, позволяющего обеспечить пуск двигателя.

Кроме того, сообщаемая на начальном этапе заряда энергия расходуется преимущественно на основной зарядный процесс. То есть на восстановление активной массы электродов. При этом процесс газообразования в аккумуляторе еще не возможен. Таким образом, заряд при постоянстве напряжения позволяет форсировать процесс заряда при подготовке батареи к эксплуатации.

Модифицированный заряд.

Такой заряд представляет собой некоторое приближение к заряду при постоянном напряжении. Цель его — несколько уменьшить силу тока в начальный период заряда и снизить влияние колебания напряжения в сети на зарядный ток. Для этого последовательно с аккумуляторной батареей в электрическую цепь включают резистор небольшого сопротивления.

Такой прием известен под названием «способа с полупостоянным напряжением». При применении данного метода напряжение на клеммах зарядного устройства поддерживается постоянным в пределах от 2,5 до 3,0 В на один аккумулятор. Считается, что для свинцовых аккумуляторов оптимальным является напряжение 2,6 В на аккумулятор, обеспечивающее заряд примерно за 8 часов.

Изменение показателей свинцового аккумулятора в процессе заряда методом модифицированного постоянного напряжения показано на схеме ниже. Эти кривые получены при напряжении 2,63 В на аккумулятор и добавочном сопротивлении 0,0091 Ом. Заряд был закончен за 8 часов.

Как видно из схемы, зарядный ток уменьшается по мере роста напряжения батареи. Рост плотности электролита начинается уже после 2-часового заряда автомобильных аккумуляторов и продолжается несколько часов. Газовыделение начинается примерно через 4,5 часа и сопровождается заметным снижением зарядного тока.

Точка на кривой изменения напряжения аккумулятора, равная 2,37 В, соответствует установке срабатывания реле напряжения, контролирующего заряд. Через 7 часов батарея получает емкость, отданную ею при разряде. Дальнейший заряд ведется заключительным режимом до небольшого перезаряда.

Изменение показателей свинцового аккумулятора в процессе заряда методом модифицированного постоянного напряжения.

Для отключения зарядного агрегата в конце заряда обычно применяют автоматические устройства. Это может быть комбинация чувствительного реле напряжения с часовым механизмом или счетчик ампер-часов. Реле напряжения запускает часовой механизм, когда напряжение на батарее достигает заданного значения. Часовой механизм отсчитывает установленное время, после чего отключает зарядное устройство.

Кроме описанных выше, применяются методы заряда автомобильных аккумуляторов, представляющие в какой-то мере их вариации. Так, в процессе длительной эксплуатации могут иметь место случаи, когда плотность электролита и степень заряженности отдельных аккумуляторов в батарее будет различной. В этом случае перед началом зимней эксплуатации целесообразно провести так называемый уравнительный заряд, который был описан выше.

Постоянный подзаряд.

Цель данного вида заряда — компенсация саморазряда. Постоянный подзаряд производят при постоянном напряжении, величина которого незначительно превышает напряжение полностью заряженной батареи.

При постоянном подзаряде нескольких батарей, включенных в электрическую цепь параллельно, в цепь каждой батареи включают переменное или постоянное сопротивление для ограничения силы зарядного тока. Главным недостатком режима постоянного подзаряда является параллельное протекание вторичного процесса. Это ускоряет коррозию решеток положительных электродов.

По материалам справочника «Аккумуляторные батареи».
Курзуков Н. И., Ягнятинский В. М.

Похожие статьи:

  • Аккумуляторные батареи с общей крышкой, устройство, соединение в батарею свинцовых аккумуляторов точечной контактной электросваркой и газовой сваркой, герметизации пластмассой.
  • Антикоры Dinitrol ML и Dinitrol 482, применение для антикоррозийной обработки днища, рамы и арок автомобиля, характеристики, свойства и недостатки, способ нанесения.
  • Как правильно прикурить автомобиль от аккумулятора другого автомобиля, схема соединения проводов для пуска двигателя автомобиля с разряженным аккумулятором.
  • Проверка работоспособности автомобильного аккумулятора, плотность электролита, измерение ЭДС, проверка разрядом на нагрузочную вилку-пробник.
  • Покупка нового автомобильного аккумулятора, критерии выбора, можно ли покупать аккумуляторную батарею большей емкости, чем штатная.
  • Как обнаружить дефекты автомобильного аккумулятора, режимы тестирования, приборы для ухода за автомобильным аккумулятором во время эксплуатации.

Понятие силы тока, ЭДС и разности потенциалов?

Электрическим током называют упорядоченное движение заряженных частиц (тел). За направление движение электрического тока условно принимают направление движения положительных зарядов. Проходящий через какую-то поверхность электрический ток характеризуется силой тока I. Сила тока является скалярной величиной, численно равная количеству электричества, проходящего через площадь S за единицу времени:

Если за любые равные промежутки времени через любое сечение проводника проходит одинаковое количество электричества с неизменным направлением зарядов, то такой ток называется постоянным:

Сила тока в Международной системе единиц (СИ) является основной и носит название Ампер. Из уравнения (1а) следует определение единицы заряда:

В системе СГС сила тока измеряется в СГСI, согласно (1а) получим:

Распределение электрического тока по сечению проводника характеризуют плотностью тока, которую можно выразить формулой:

В случае постоянного тока его плотность будет одинакова и равна:

Плотность тока j является векторной величиной, направленной вдоль тока и численно  равная количеству электричества, протекающему через единицу площади, ориентированной перпендикулярно направлению протекания тока, за единицу времени, в системе СИ плотность тока измеряют в А/м2.

Важно отметить, что различают несколько видов электрического тока. Предположим, что в пространстве перемещается какое-то заряженное макроскопическое тело (шар, например). Поскольку вместе с этим телом будут перемещаться и заряды, то возникнет направленное движение электрических зарядов – электрический ток. Электрический ток, связанный с движением заряженных макроскопических тел называют конвекционным.

Если огромное количество заряженных частиц упорядоченно перемещаются внутри какого-нибудь тела вследствие того, что в нем создано электрическое поле, то данное явление будет носить название ток проводимости. Для его получения необходимо наличие источника тока и  замкнутой цепи. Вектор напряженности поля Е имеет направление от положительного заряда к отрицательному. Отсюда следует, что находящиеся внутри проводника отрицательные заряженные частицы будут двигаться против поля, а положительные – по полю.

Если электрические заряды движутся под влиянием внешнего поля в вакууме, то данное явление называют электрический ток в вакууме.

Более детально остановимся на отдельных закономерностях, которые больше характерны для тока проводимости.

Представим, что на концах определенного проводника длиной l существует разность потенциалов Δφ = φ1 – φ2, которая создает внутри этого проводника электрическое поле Е, направленное в сторону падения потенциала (рисунок ниже):

Согласно формуле:

При этом в проводнике возникнет электрический ток, который будет идти от большего потенциала (φ1) к меньшему (φ2).

Движение зарядов от φ1 к φ2 приводит к выравниванию потенциалов во всех точках. При этом в проводнике исчезает электрическое поле, и протекание электрического тока прекращается. Отсюда следует, что обязательным условием существования электрического тока является наличие разности потенциалов Δφ = φ1 – φ2 ≠ 0, а для его поддержания необходимо специальное устройство, которое будет поддерживать данную разницу потенциалов. Это устройство называют источник тока.

В качестве источников тока могут использовать электрические генераторы, аккумуляторы, термоэлементы и гальванические элементы. Источник тока также выполняет еще одну задачу – замыкает электрическую цепь, по которой и осуществляется непрерывное движение заряженных частиц. Электрический ток протекает по внутренней части – источнику тока, и внешней – проводнику. В источнике тока имеется два полюса – положительный с более высоким потенциалом и отрицательный с более низким потенциалом. При разомкнутой внешней цепи на положительном полюсе источника образуется недостаток электронов, а на отрицательном наоборот – переизбыток. В источнике тока разделение зарядов производят с помощью сторонних сил – направленных против кулоновских сил, действующих на разноименные заряды в проводниках самого источника тока. Сторонние силы могут иметь самое различное происхождение – химическое, биологическое, тепловое, механическое и другое.

Если электрическая цепь замкнута, то по ней протекает электрический ток и при этом совершается работа сторонних сил. Данная работа складывается из работы, совершаемой внутри самого источника тока против сил электрического поля (Аист), и работы, совершаемой против механических сил сопротивления среды источника (А/), то есть:

Электродвижущая сила источника тока – это величина, которая равна отношению работы, совершаемой сторонними силами при перемещении положительного точечного заряда вдоль всей электрической цепи, включая и источник тока, к заряду:

По определению работа против сил электрического поля равна:

А/ = 0 если полюсы источника разомкнуты, и тогда из формулы (5) следует:

Отсюда следует, что электродвижущая сила источника тока при разомкнутой внешней цепи будет равна разности потенциалов на его полюсах.

Заряд, ток и разница потенциалов

Символы схем — вы встречали эти символы схем в GCSE Physics.

Обычный ток течет по цепи от положительной (+) стороны ячейки к отрицательной (-). Однако электронов протекают по цепи в противоположном направлении от отрицательной (-) стороны ячейки к положительной (+).

Заряд, ток и разность потенциалов

Заряд (Q) — заряд измеряется в кулонах (Кл).

  • Один электрон несет заряд 1,6 x 10 -19 Кл.

Ток (I) — измеряется в амперах (А).

  • Ток — это скорость потока заряда . Ток в 1 А означает, что 1 кулон заряда проходит через точку в цепи каждую секунду. (1 A = 1 C s -1 ) Ток в цепи измеряется с помощью амперметра, который включен последовательно с интересующим компонентом в цепи.
  • I = ток в амперах, А
  • DQ = заряд в кулонах, К
  • Dt = время в секундах, с

Разность потенциалов (В) — измеряется в вольтах (В).

  • Потенциальная разница — это работа, выполненная на единицу оплаты . Разность потенциалов в 1 В означает, что на один кулон заряда выполняется 1 джоуль работы. (1 В = 1 Дж C -1 ) Разность потенциалов в цепи измеряется с помощью вольтметра, который устанавливается параллельно с интересующим компонентом в цепи.
  • В = разность потенциалов в вольтах, В
  • Вт = выполненная работа или переданная энергия в джоулях, Дж
  • Q = заряд в кулонах, Кл

Сопротивление (Вт) — это отношение разности потенциалов на компоненте к току, протекающему через него, измеряется в омах (Вт).

  • R = сопротивление в Ом, Вт

  • В = разность потенциалов в вольтах, В

  • I = ток в амперах, А

Примеры;

Q1) Если все электроны несут заряд 1,6 x 10 -19 C, сколько электронов потребуется, чтобы получить общий заряд в один кулон?

Q2) Если ток 0.50 ампер протекает через цепь в течение 120 секунд. Сколько заряда будет передано в компонент в цепи?

Q3) Заряд в 4,0 кулонов был перемещен через разность потенциалов 24 вольта, сколько энергии было передано?

Q4) Разность потенциалов на компоненте составляет 12 В, а ток через него составляет 0,37 А. Каково сопротивление компонента?

2.3: Заряд и ток — Engineering LibreTexts

Как уже отмечалось, заряд — это сила притяжения. Обозначается буквой \ (Q \) и имеет единицы кулонов. Электроны заряжены отрицательно, а протоны — положительно. Все электроны и протоны обладают одинаковой величиной заряда, примерно 1,602E − 19 кулонов. Таким образом, один кулон эквивалентен заряду примерно 1 / 1,602E − 19 или 6,242E18 электронов. Кроме того, противоположные заряды притягиваются, в то время как одинаковые заряды отталкиваются, подобно полюсам магнита.

Можно перемещать заряд из одной точки в другую. Скорость движения заряда во времени называется текущей. Обозначается буквой \ (I \) и имеет единицы ампер (или ампер) 1 . Один ампер тока определяется как один кулон в секунду.

\ [1 \ text {amp} \ Equiv 1 \ text {coulomb} / 1 \ text {second} \ label {2.1} \]

То есть, один усилитель можно представить как примерно 6,242E18 электронов, проходящих через провод за период в одну секунду. Рассмотрим рисунок 2.3.1 .

Рисунок 2.3.1 : Определение тока как заряда, протекающего через провод.

Здесь у нас есть провод, по которому в направлении стрелки протекают электроны. Эту проволоку перерезаем с воображаемой плоскостью, оставив нам подсвеченный диск. Теперь представьте, что мы можем подсчитать количество электронов, проходящих через этот диск за одну секунду. Поскольку мы знаем заряд, которым обладает один электрон, мы просто умножаем количество электронов на заряд каждого, чтобы получить общий заряд, и таким образом мы приходим к току.В виде формулы:

\ [I = Q / t \ ​​label {2.2} \]

где

  • \ (I \) — ток в амперах,
  • \ (Q \) — заряд в кулонах,
  • \ (t \) — время в секундах.

Распространенная аналогия электрического тока — это поток воды по трубе или реке. Точно так же, как мы можем представить поток воды как «галлоны или литры в минуту», мы представляем электрический ток как «кулоны в секунду».

Пример 2.3.1

За полсекунды определенный аккумулятор обеспечивает заряд в три кулона.Определите результирующий ток.

Решение

\ [I = \ frac {Q} {t} \ nonumber \]

\ [I = \ frac {3C} {0,5 с} \ nonumber \]

\ [I = 6 A \ nonumber \]

Пример 2.3.2

Устройство выдает ток 25 мА. Определите заряд, переносимый за две секунды, вместе с эквивалентным общим количеством электронов.

Решение

\ [I = \ frac {Q} {t} \ nonumber \]

\ [Q = I \ times t \ nonumber \]

\ [Q = 25 мА \ раз 2 с \; \ text {из определения \ ref {2.1}, ампер-секунды — кулоны} \ nonumber \]

\ [Q = 50 мКл \ nonumber \]

Так как один кулон эквивалентен 6,242E18 электронов, просто умножьте, чтобы найти общее количество переданных электронов.

\ [\ text {Общее количество электронов} = Q \ times \ text {количество электронов на кулон} \ nonumber \]

\ [\ text {Всего электронов} = 50 мкКл \ умножить на 6.242E18 \ nonumber \]

\ [\ text {Всего электронов} = 3.121E17 \ text {электроны} \ nonumber \]

В итоге, чем больше заряд, переданный в течение заданного периода времени, тем больше ток.Современные электрические и электронные системы могут работать с токами ниже пикоампера или, в другом крайнем случае, до тысяч ампер. Это потрясающий диапазон. Это примерно эквивалентно одной капле воды, капающей из протекающего крана каждую секунду, по сравнению с 1000-кратным потоком воды над Ниагарским водопадом.

Список литературы

1 \ (I \) означает \ (I \) ntensity (тока), и был назван так Андре-Мари Ампером.

Заряд и ток

Заряд и ток
Далее: Напряжение и работа Up: Копаем глубже Предыдущий: Единицы СИ

Частицы с электрическим зарядом действуют друг на друга.Величина этой силы зависит от заряда на каждая частица. Величина силы обратно пропорциональна пропорционально квадрату расстояния между частицы.

Базовая единица заряда — кулон . Один кулон равен заряду электроны. Другими словами, у одного электрона есть заряд из кулоны. Символ для заряд есть или.

Электрические цепи перемещают электрический заряд так, что полезная работа сделана.Эти движущиеся заряды генерируют электрический ток , который мы обозначаем как или. В другими словами, если — это сумма заряда на конкретная точка в пространстве во времени, затем текущая проходящая через эту точку равна первой производной по времени из . Другими словами,


Базовая единица измерения тока — ампер (обозначена сокращение). Один ампер равен одному кулону заряд проходит через точку в пространстве за одну секунду. В другими словами, один ампер равен одному кулону в секунду.

Рассмотрим провод, по которому проходит ток в амперы. через определенную точку на этом проводе. Заряд может либо двигаться справа налево, либо слева направо. Так чтобы полностью уточнить природу тока, мы должны также укажите направление, в котором течет ток. путешествия. Это делается путем связывания знака с электрический ток. Другими словами, текущий — это со знаком количество.

Знак, данный току, зависит от того, кто мы заинтересован в измерении.Движущийся заряд можно представить как либо

  • отрицательно заряженные электроны, движущиеся по проводу,
  • или положительно заряженные частицы, движущиеся через провод
В первом случае мы имеем так называемый электронов. текущий . Во втором случае имеем так называемый условный ток . Обычной практикой является использование обычные, а не электронные токи. На протяжении всего нашего работать, это соглашение, которое мы будем использовать.

На принципиальных схемах мы обозначаем ток, текущий в элемент схемы стрелкой, обозначающей один из терминалы.Стрелка обычно обозначается размером электрический ток. Стандартное соглашение (называемое пассивным соглашение о маркировке), используемое при маркировке этих стрелок, предназначено для используйте положительное число, когда ток давит положительный заряжается в устройство. Если число отрицательное, то ток вытягивает положительные заряды из устройства. Рисунок 16 иллюстрирует соглашение о пассивной маркировке резистора.

Рисунок 16: Ток, протекающий в элементе цепи

Напомним, что номер, обозначающий текущий, подписан Стрелка.Это означает, что мы можем получить две разные метки для то же направление обычного тока. Фигура 17 показаны две такие метки. В в первом случае мы выталкиваем положительный заряд из терминал в устройство. Во втором случае мы вытягивая положительный заряд из устройства в терминал. Конечный результат для обеих этикеток одинаковый, а именно: поток положительных зарядов идет слева направо через устройство.

Рисунок 17: Две разные маркировки одного и того же тока

Полный заряд, поступающий в элемент схемы, получается следующим образом: интегрируя дифференциальное уравнение.Предположим, что заряд изначально есть, затем общий заряд, поступающий в устройство между временами и будет




Далее: Напряжение и работа Up: Копаем глубже Предыдущий: Единицы СИ
Майкл Леммон 2009-02-01

Заряд и ток — Заряд, ток и напряжение — CCEA — GCSE Physics (Single Science) Revision — CCEA

Электрический ток — это поток заряженных частиц.

В металлических проводниках заряженными частицами являются свободные электроны.

Электроны могут свободно перемещаться от одного иона к другому, и чистый поток этих электронов в одном направлении представляет собой электрический ток.

Для движения свободных электронов в одном направлении требуется источник энергии, например элемент или батарея.

Charge

Электроны — это отрицательно заряженные частицы, которые передают электрическую энергию от элемента через проводящие провода в виде электрического тока.

Заряд измеряется в кулонах , C.

Заряд электрона составляет 1,6 x 10 -19 C.

Другими словами, требуется 6,250,000,000,000,000,000 электронов, чтобы составить 1 кулон заряда.

Кулон заряда — это просто очень большая группа электронов.

Связь между током I и количеством заряда Q

Электрический ток — это поток заряженных частиц.

Величина электрического тока — это скорость протекания заряда.

Ток I = \ (\ frac {\ text {количество заряда Q}} {\ text {время t}} \)

I = \ (\ frac {\ text {Q}} {\ text {t }} \)

Его часто вспоминают как:

Количество заряда Q = ток I x время t

Q = It

Где:

Q = количество заряда в кулонах, C

I = ток в амперы, А

t = время в секундах, с

I = \ [\ frac {\ text {Q}} {\ text {t}} \] I = Q ÷ t
Q = It Q = I xt
t = \ [\ frac {\ text {Q}} {\ text {I}} \] t = Q ÷ I

Один ампер — это ток, который протекает, когда один кулон заряда проходит через точку в цепи за одну секунду.

Ток и заряд | IOPSpark

Идеи начального уровня

На вводном уровне описания того, что происходит в электрических цепях, просто качественные. не уместно обсуждать концепции количественно.

Идеи среднего уровня

Определение тока

Ток можно описать как поток заряда, измеряемый в кулонах.Затем вы описываете и определяете кулон с точки зрения меднения. Вы даже можете указать, что единичный ток, один ампер (или ампер), означает один кулон в секунду с точки зрения меднения (0,000 000 329 кг меди переносится каждую секунду в ванне для меднения). Хотя это не согласуется с нынешней модой определения токов с помощью сил, это дает студентам гораздо более простой способ изображения токов. У них уже есть, насколько известно, сильное чувство токов как потоков маленьких электронов, и если вы сгруппируете эти электроны в большие кулоны заряда, вы легко сможете убедить их подумать о токах, измеряемых в кулонах в секунду.

Определение разницы потенциалов

Когда учащиеся поймут, как переносится энергия, можно четко обсудить разность потенциалов и определить вольт как джоуль на кулон. Обсуждение источников питания как источников энергии и электрических зарядов как носителей энергии помогает новичку понять, почему ток в последовательной цепи не уменьшается, когда он протекает через компоненты передачи энергии, такие как лампы. Вы можете рассматривать разность потенциалов как фундаментальную измеримую величину, описываемую как перенос энергии для каждого кулона, проходящего через рассматриваемую область; е.грамм. энергия, передаваемая от батареи к лампе и, следовательно, в окружающую среду.

Конечно, это ненаучная фантастика — изображать кулоны, несущие на своей спине груз энергии и выбрасывающие часть нагрузки в каждой части цепи, а затем собирающие новую нагрузку каждый раз, когда они проходят через батарею. Тем не менее, если вы время от времени предупреждаете студентов, что это искусственная картина с нереалистичными деталями, они могут использовать модель, чтобы получить полезное представление о разнице потенциалов.

Тогда сопротивление, которое может быть более удобным при разработке профессиональной схемы электрических блоков, занимает второстепенное место как [разность потенциалов] / [ток], а один ом просто определяется как название для одного вольт / ампер. Это просто словарная работа.

С этими описаниями и определениями разности потенциалов и тока очевидно, что разность потенциалов x ток дает нам мощность, скорость, с которой передается энергия.На сленге «вольт x = ватт, ».

А когда вы генерируете э.д.с. вы также можете дать четкое описание этой концепции.

Идеи для продвинутого уровня

В более формальных трактовках электричества единичный ток выбирается в качестве фундаментальной величины (определяемой в терминах силы между параллельными токами). Сопротивление — это полезная производная величина, вторичный стандарт, который можно легко сохранить и скопировать. Тогда единица разности потенциалов выводится из единиц заряда и энергии (или тока и мощности).

Какой бы удобной ни была эта схема, она оставляет без четкого описания саму природу разности потенциалов. Конечно, на вводном уровне студенты находят «напряжение» загадочным понятием, часто неопределенно описываемым как электрическое давление и часто описываемым как умножение тока на сопротивление. Когда использование разности потенциалов распространяется на случаи, когда нет тока, или случаи, когда нет сопротивления по закону Ома, это остается очень загадочным.

Развитие электрических знаний — от начального до продвинутого уровня

Здесь существует опасность путаницы между несколькими различными целями при построении электрических знаний.Это вопрос тщательного определения основных единиц и получения вторичных единиц; это касается курсов продвинутого уровня. Это вопрос описания и определения физических величин, которые будут измеряться в этих единицах. Здесь вам нужно знать физическое соотношение, извлеченное из экспериментов, например, теплопередача изменяется, как ток 2 , или скорость меднения изменяется как ток . Существуют «рабочие» определения в техническом смысле этого слова, которые описывают схему измерения с точки зрения реального оборудования, которое может быть использовано.

Раньше ученые иногда использовали понятия, которым нельзя было дать рабочее определение. В настоящее время они более осторожны и пытаются определить или, по крайней мере, описать концепции физических величин в терминах возможных или, по крайней мере, мыслимых методов их измерения. Такие определения должны давать четкое представление о концепции; но они не всегда приводят к наиболее удобной единице измерения физической величины. Выбранная единица может быть определена совершенно отдельно — вы часто обнаруживаете, что она была выбрана ранее в истории предмета.

Нет никаких логических возражений против определения единицы тока в терминах массы меди, осажденной за секунду при электролизе, хотя ток формально измеряется в терминах силы между проводами или катушками, по которым протекает ток.

Заряд, ток и напряжение — Теория Лабстера

В этом разделе мы опишем, что такое заряд, ток и напряжение, и каковы их основные характеристики.

Заряд связан с равновесием между протонами и электронами. Электроны несут отрицательный заряд, а протоны несут положительный заряд. Наименьший возможный заряд — это заряд, переносимый электроном, который равен 1,6×10 -19 кулонов (Кл). 1 Кулон — это количество заряда, переносимого 6,25×10 18 электронами. Итак, чтобы упростить это, мы могли бы сказать, что когда мы говорим об 1 кулоновском заряде, это в основном означает заряд, переносимый этим количеством электронов.

Ток (или сила тока) — это движение электрически заряженных частиц. Такие частицы, как электроны, имеют электрический заряд, поэтому, когда электроны движутся через проводник (например, кабель), они создают электрический ток. Единицей измерения силы тока является ампер (А). 1 Ампер можно определить как 1 кулон в секунду, что означает, что в определенной точке (например, кабеля) 1 кулон заряда проходит через определенную точку кабеля каждую секунду.

Напряжение можно определить как разность электрических потенциалов между двумя разными точками. Электрический потенциал эквивалентен потенциальной энергии электричества. По сути, напряжение может заставить электрически заряженные частицы (т.е.электроны) двигаться (преобразовывать потенциальную энергию в кинетическую), и, поскольку движение электрически заряженных частиц может быть определено как ток , напряжение может вызывать ток. Напряжение измеряется в вольтах (В). 1 Вольт можно определить как 1 Джоуль на кулон.Джоуль — это единица измерения работы (работа, конечно, в рамках концепции физики), являющаяся измерением передачи энергии. В этом случае ток 9 вольт будет получать 9 джоулей энергии на кулон заряда.

При работе с цепями ток можно понимать как количество электронов, проходящих через точку за период времени, а напряжение — это количество энергии, передаваемой этим электронам. В то время как напряжение (энергия) используется электрическими компонентами цепи (лампочка, резистор и т. Д.)), тока нет, электроны не исчезают, ток постоянный, так как количество электронов, проходящих по кабелю, всегда одно и то же, а меняется энергия в них (напряжение).

Рис. 1. Визуальное представление заряда, тока и напряжения.

электрических цепей

Поток заряда

Электрический ток — это поток заряда, так же как потоки воды — это поток молекул воды.Молекулы воды имеют тенденцию течь из областей с высокой потенциальной гравитационной энергией в области с низкой потенциальной гравитационной энергией. Электрические токи текут от высокого электрического потенциала к низкому электрическому потенциалу . И чем больше разница между высоким и низким потенциалом, тем больше тока течет!

В большинстве электрических токов движущиеся заряды представляют собой отрицательные электроны. Однако, по историческим причинам, восходящим к Бену Франклину, мы говорим, что обычных токов, протекающих в направлении положительных зарядов, переместятся на .Хотя это неудобно, довольно легко оставаться прямо, если вы просто помните, что действительные движущиеся заряды, электроны, текут в направлении, противоположном направлению электрического тока. Имея это в виду, мы можем утверждать, что положительный ток течет от высокого потенциала к низкому потенциалу , даже несмотря на то, что носители заряда (электроны) фактически текут от низкого потенциала к высокому.

Электрический ток (I) измеряется в амперах (A), или амперах, и может быть рассчитан путем нахождения общего количества заряда (∆q) в кулонах, которое проходит через определенную точку за данный момент времени (t). .Следовательно, электрический ток можно рассчитать как:

Вопрос: Заряд 30 кулонов проходит через Резистор 24 Ом за 6,0 сек. Что ток через резистор?

Ответ:

Вопрос: Заряд, протекающий со скоростью 2,50 × 10 16 элементарных зарядов в секунду, эквивалентен току

  1. 2,50 × 10 13 A
  2. 6.25 × 10 5 А
  3. 4,00 × 10 –3 A
  4. 2,50 × 10 –3 A

Ответ:

Вопрос: 1,5-вольтовый элемент AAA подает 750 миллиампер тока через лампочку фонарика в течение 5 минут, а 1,5-вольтовый элемент C подает 750 миллиампер тока через ту же лампочку фонарика в течение 20 минут. По сравнению с общим зарядом, передаваемым элементом AAA через лампочку, общий заряд, передаваемый элементом C через лампу, составляет

  1. вдвое меньше
  2. вдвое больше
  3. то же
  4. в четыре раза больше

Ответ: (4) Если Δq = It, и обе ячейки дают 0.750A, но элемент C обеспечивает тот же ток в четыре раза дольше, он должен обеспечивать в четыре раза больше общего заряда по сравнению с элементом AAA.

.

alexxlab

Добавить комментарий

Ваш адрес email не будет опубликован. Обязательные поля помечены *